SlideShare ist ein Scribd-Unternehmen logo
1 von 55
Downloaden Sie, um offline zu lesen
Page 1 of 55
GEN ED SET B
1. Alin ang kahulugan ng AGAW-BUHAY?
A. Masiglang-masigla
B. Malapit nang mamatay
C. Pagpapatuloy ng buhay
D. Mahirap na buhay
2. Robert Frost wrote the poemAcquainted with the Nightfrom which the stanza is taken:
I have been one acquainted with the night.
I have walked out in rain-back in rain.
I have out walked the farthest city light
The poet in the stanza talks of
A. isolation and loneliness.
B. happiness in having been acquainted with the night.
C. joy getting out of the house.
D. youthful delight playing in the rain.
3. The following are defects present at the time of marriage which is voidable and annullable
EXCEPT
A. impotence
B. deceit
C. fraud
D. threat
4. What does a stick of unlit dynamite demonstrate?
A. Chemical Energy
B. Nuclear Energy
C. Kinetic Energy
D. Potential Energy
Page 2 of 55
5. Ang sumusunod ay isang talumpati na may pamagat na SA KABATAAN na sinulat ni Onofre
Pagsanghan.
Isa sa mga salitang napag-aralan natin sa wikang Pilipino ay ang salitang NABANSOT.
Kapang ang isang bagay daw ay dapat pang lumaki ngunit ito'y tumigil na sa paglaki, ang
bagay na ito raw ay NABANSOT. Marami raw uri ng pagkabansot ngunit ay pinakamalungkot
na uri raw ay ang pagkabansot ng isipan, ng puso at ng diwa.
Ang panahon ng kabataan ay panahon ng paglaki, ngunit ang ating paglaki ay kailangang
paglaki, at pag-unlad ng ating buong katauhan, hindi lamang ng ating sukat at timbang. Kung
ga-poste man ang ating taas at ga-pison man ang ating bigat ngunit kung ang pag-iisip
naman nati'y ga-kulisap lamang kay pangit na kabansutan. Kung tumangkad man tayong
tangkad-kawayan at bumilog man tayong bilog-tapayan, ngunit kung tayo nama'y tulad
nibondyingay di mapagkatiwalaan-anong laking kakulangan. Kung magkakatawan tayong
katawangTarzanat mapatalas ang ating isipang sintalas ng kay Rizal, ngunit kung ang ating
kalooban nama'y itim na duwende ng kasamaan-anong kapinsalaan para sa kinabukasan.
Kinabukasan, kabataan, tayo raw ang pag-asa ng Inang Bayan. Tayo raw ang maghahatid sa
kanya sa langit ng kasaganaan at karangalan o hihils sa kanya sa putik ng kahirapan at
kahihiyan. Ang panahon ng pagkilos ay ngayon, hindi bukas, hindi sa isang taon. Araw-araw
ay tumutuwid tayong palangit or bumabaluktod tayong paputik. Tamang-tama ang sabi ng
ating mga ninunong kung ano raw ang kinamihasnan ay siyang pagkakatandaan. Huwag
nating akalaing makapagpapabaya tayo ng ating pag-aaral ngayon at sa araw ng bukas ay
bigla tayong magiging mga dalubhasang magpapaunlad sa bayan. Huwag nating akalaing
makapaglulublob tayo ngayon sa kalaswaaan at kahalayan at sa mahiwagang araw ng bukas
sigla tayong magiging ulirang mga magulang.
Kabataan, tunay na pag-ibig sa bayan, ang tunay na nasyonalismo, ay wala sa tamis ng
pangarap wala rin sa pagpag ng dila. Ang tunay na pag-ibig ay nasa pawis ng gawa.
Alin sa mga sumusunod ang mensahe ng talumpati?
A. Ang mataas na paniniwala at taimtim na pananalig ay kailangang taglayin upang ang
hangarin sa buhay ay ating kamtin.
B. Ang panahon ng kabataan ay panahon ng paglaki at pagbabagong
makabuluhan.
C. Ang gawa ang siyang sukat ng kadakilaan.
D. Ang kabataan ay siyang pag-asa ng bayan.
6. Ano ang kahulugan ng taludtod na ito?
Ang anak man ay alagaan sa marubdob na pagsuyo sikapin ma sa sarili'y huwag siyang
maging luko talipandas sa paglaki na sa sama marahuyo sa lahi mo't sa Bayan may isang tinik
sa balaho.
A. Mahalin ang anak ng walang hangganan.
B. Tamang pagpapalaki sa anak ang dapat.
Page 3 of 55
C. Suyuin ang anak at ibigay lahat ng hilig.
D. Paligayahin ang tahanan.
7. The carat is a unit of measure used to weigh precious stones. It equals 3.086 grains. How
many grains does a 2.8 carat diamond weigh?
A. 864.08
B. 86.408
C. 8.6408
D. 8640.8
E. none of these
8. Alin ang kahulugan ng KAHIRAMANG SUKLAY?
A. Kakilala
B. Kaibigan
C. Karibal
D. Kalahi
9. Nasa anong kaganapan ng pandiwa ang pangungusap?
Naglaro ng basketball sa Rizal Stadium ang koponan ng aming pamantasan.
A. Sanhi
B. Tagaganap
C. Kagamitan
D. Ganapan
10. Julieta Villaruel was a landowner from Cabiao, Nueva Ecija. Under the CARL, she was claiming
8 hectares, 5 hectares of which represented the retention limit and the 3 hectares for her only
child. Why was her child denied the 3 hectares?
A. Her son was 15 years old who was actually tiling the farm
B. Her son was 17 years old who was managing the farm
C. Her son was 13 years old who has been helping till the farm
D. Her son was 19 years old who was actually tilling or managing the farm
Page 4 of 55
11. Sa Espiritu ni Bathala ang nangangalaga ng kanilang kalusugan ang ipinahihiwatig na katangian ay
__________.
A. malinis
B. mabisa
C. maliksi
D. makapangyarihan
12.Find m in the proportion m/12 = 30/24.
A. 30
B. 15
C. 20
D. 25
E. none of these
13. One of the most outstanding accomplishments of the cooperative movement is the encouragement
of thrift. Which maxim of God puts this into practice?
A. God helps those who help themselves
B. Look at the birds: they do not plant seeds,/gather a harvest and put it in barns; yet your
Father in heaven takes care of them!
C. He is near to those who call to Him, who call to Him with sincerity
D. Happy are those who are merciful to others; God will be merciful to them!
14. Which one is the human right to life?
A. Peace
B. Live in national and international order
C. Own property
D. Fair trial
15. Which antidote would have a similar effect if vinegar or citrus juice were not available?
A. Milk
B. Raw egg white
Page 5 of 55
C. Vegetable oil
D. Water
16. Thousands of street children in large Brazilian cities were murdered by paramilitary death squads
which included police officers. What could be the reason why these operations were not suppressed by
the government?
A. Totalitarian governments do not give protection to their people.
B. The business people even funded these operations to clean up their streets and
neighborhoods.
C. The Universal Declaration of Human Rights was only lip service.
D. To how they treat their people was nobody else's business.
17. What is the difference between the largest 4-digit number and the smallest 4-digit number?
A. 8999
B. 8000
C. 9998
D. 8888
E. none of these
18. Si Mariano Ponce ay propagandistang may sagisag sa panulat na
A. Tamaraw
B. Tikbalang
C. Kapre
D. Kalapate
19. Below is a biographical sketch.
Success often comes to those with humble beginnings, Elvis Aron Presley was born on January 8,
1935, in Tupelo, Mississippi. He first sang in a church choir and taught himself to play the guitar, but
he never learned to read music. By 1953, he had moved to Memphis, Tennessee, graduated from high
school, and enrolled in night school to become an electrician. That year, at Sun Records, Presley
recorded a personal record for his mother, a song that was heard by the company's president. As a
result of the president's recognition, Presley's first record,That's all Right, Mama,was out in 1954.
He toured the South, and in 1955 five of his records were released simultaneously. His first national
television appearance was that year on Jackie Gleason'sThe Stage Show,but Presley became known
for his appearance onThe Ed Sullivan Show,where the young singer gyrated as he sangrock n'
Page 6 of 55
rollmusic. During the live television performance, Presley was photographed only from the waist up
because his motions were considered obscene.
Elvis the Pelvisbegan his film career in 1956 with LOVE ME TENDER and signed a long term film
contract. The movie critics were not always kind, but teenagers flocked to Presley's films. Within a few
short years, Presley had established a career that would span twenty-five years of ups and downs and
make him one of the most popular entertainers in history. Long after his untimely death at age 42,
Presley would be remembered asThe King of Rock n' Roll
The main idea of the sketch is that
A. singers are more successful if they appear in films
B. there has always been obscenity on television
C. opportunity and luck are often as important as hard work
D. celebrities are usually more famous after their death
20. What do the following lines CONVEY?
Midnight, not a sound from the pavement.
Has the moon lost her memory?
She is smiling alone.
In the lamp light the withered leaves
Collect at my feet
And the wind begins to moan
A. Confusion
B. Optimism
C. Loneliness
D. Eagerness
21. Anong uri ng panghalip ang salitang may salungguhit sa pangungusap?
May padalang tulong ang pamahalaan para sa kanila.
A. Pambalana
B. Palagyo
C. Paari
D. Palayon
22. Alin ang di-karaniwang anyo ng pandiwang WINIKA KO?
A. Ikako
Page 7 of 55
B. Wikako
C. Kako
D. Wika ko
23. Why does a pendulum in a grandfather clock once set in motion continue to swing, thereby
regulating the clock's movement? This is due to the Law of
A. Universal Gravitation
B. Action and Reaction
C. Applied Force
D. Inertia
24. Which of the following BEST demonstrates the greenhouse principle?
A. A heated aquarium
B. A car with rolled-up windows
C. A microwave oven
D. A solar battery -powered calculator
25. Which one explains why oxygen a gas, is the largest component of the Earth’s crust?
A. Oxygen gives Earth's crust its lightness
B. Oxygen is the most abundant element in the world
C. Oxygen is capable of combining with most of the elements in the Earth's crust.
D. Oxygen is needed to sustain all life on Earth.
26. Which is the BEST WAY to write the underlined portion of this sentence?
Many viewers taped shows to watch later.
A. tapped
B. had taped
C. tape
D. had tapped
Page 8 of 55
27. Below is an excerpt from John F. Kennedy's "Inaugural Address."
In your hands, my fellow citizens, more than mine, will rest the final success or failure of our course.
Since this country as founded, each generation of Americans has been summoned to give testimony to
its national loyalty. The graves of young Americans into answered the call to service surround the
globe.
Now the trumpet summons us again-not as a call to bear arms, though, arms we need, not as a call to
battle, though embattled we are; but a call to bear the burden of a long twilight struggle, year in and
year out, rejoicing in hope, patient in tribulation, a, struggle against the common enemies of man:
tyranny, poverty, disease, and war itself.
Can we forge against these enemies a grand and global alliance, North and South, East and West, that
can assure a more fruitful life for all mankind? Will you join me in this historic effort?
In the long history of the world, only a few generations have been granted the role of defending
freedom in its hour of maximum danger. I do not shrink from this responsibility; I welcome it I do not
believe that any of us would exchange places with any other people or any other generation. The
energy, the faith, the devotion which we bring to this endeavor will light our country and all who serve
it, and the glow from that fire can truly light the world.
And so, my fellow Americans, ask not what your country can do for you; ask what you can do for your
country.
My Fellow citizens of the world ask not what America will do for you, but what together we can do for
the freedom of man.
The tone of the speech can BEST be characterized as
A. the use of the personal pronouns we and us to build rapport with listeners
B. catchy turns of phrase in which subjects and objects are inverted
C. a standard, predictable rhythm and the use of rhyme
D. the repetition of key words
28. Sa alin salita magkakaroon ng saglit na paghinto kung pinapilitang si Rose ang nakabasag ng
pinggan?
Hindi si Rose ang nakabasag ng pinggan.
A. Rose
B. Hindi
C. Nakabasag
D. Pinggan
29. Below is the poem written by Edgar Lee Masters in 1915:
What Happened to Tom Merritt?
At first I suspected something
She acted so calm and absent-minded.
And one day I heard the back door shut,
Page 9 of 55
As I entered the front, and I saw him slink
Back of the smoke house into the lot,
And across the field.
And I meant to kill him on sight.
But that day, walking near Fourth Bridge,
Without a stick or a stone at hand,
All of a sudden I saw him standing,
Scared to death, holding his rabbits,
And all l could say was,Don't, Don't, Don't,
As he aimed and fired at my heart.
The way in which the poet presents these words in line 12 implies that Tom
A. tried to annoy the other man.
B. was shot before the finished the statement.
C. did not want to hurt the other man.
D. begged the man to stop seeing his wife.
30. Carter's part in relinquishing U.S. control of the Canal Zone to Panama is described as a victory for
__________.
A. conservatism
B. anti-imperialism
C. isolationism
D. imperialism
31. How many whole numbers can divide 30 exactly?
A. Eight
B. Six
C. Five
D. Four
E. none of these
32. Which are the next three terms in the progression 1, 4, 16 . . . 8 terms?
A. 64, 256, 1024
B. 67, 259, 1027
C. 66, 258, 1026
Page 10 of 55
D. 65, 257, 1025
E. none of these
33. Which is NOT personal integrity?
A. Time
B. Place
C. Order
D. Harmony
34. One package is 100 pounds, and the other is 150 pounds. The weight of the second package is
how many times that of the first?
A. 1½ times heavier
B. ½as heavy
C. none of these
D. 10 pounds heavier
E. 20 pounds heavier
35. If a baseball player hits 10 home runs in the first 45 games, at the same rate how many home
runs can he expect to hit during the 162-game season?
A. 38
B. 42
C. 36
D. 40
E. none of these
36. Of the following changes in the socio-economic, political cultural and physical that have occurred
in the Filipino family, which one remains to be TRUE?
A. The loss of the traditional evening prayer and the ritual of blessing (mano)
B. The unity of the family despite competing demands
Page 11 of 55
C. The continued parental influence over children's language dress and other behavior
D. The continued support for parents and siblings
37. What day follows the day before yesterday if 2 days from now will be Sunday?
A. Tuesday
B. none of these
C. Saturday
D. Wednesday
E. Thursday
38. What correction should be made to this sentence?
One of their theories is that the first child receives more of the parents' attention than other children
so first-borns tend to be more intellectual.
A. Change is to are
B. Insert a comma after children
C. Change parents' to parent's
D. Change theories to theory's
39. The poem below is entitledSuburban Prophecywhich is written by Howard Nemerov.
On Saturday, the power-mowers' whine
Begins the morning. Over this neighbourhood
Rises the keening, petulant voice, begin
Green oily teeth to chatter and munch the cud.
Monsters, crawling the carpets of the world,
Still send from underground against your blades
The roots of things battalions green and curled
And tender, that will match your blades with blades
Till the revolted throats shall strangle on.
The tickle of their dead, till straws shall break
Crankshafts like camels, and the sun go down
On dinosaurs in swamps. A night attack
Follows and by the time the Sabbath dawns
All armored beasts are eaten by their lawns.
How long does the action of the poem take place?
Page 12 of 55
A. A week
B. Twenty-four horns
C. An afternoon
D. A morning
40. A meter was cut at the 35-cm mark. What is the ratio of the smaller piece to the larger piece?
A. 7:13
B. 65:35
C. 35:100
D. 65:100
E. none of these
41. Below is an excerpt from John F. Kennedy's "Inaugural Address."
In your hands, my fellow citizens, more than mine, will rest the final success or failure of our course.
Since this country as founded, each generation of Americans has been summoned to give testimony to
its national loyalty. The graves of young Americans into answered the call to service surround the
globe.
Now the trumpet summons us again-not as a call to bear arms, though, arms we need, not as a call to
battle, though embattled we are; but a call to bear the burden of a long twilight struggle, year in and
year out, rejoicing in hope, patient in tribulation, a, struggle against the common enemies of man:
tyranny, poverty, disease, and war itself.
Can we forge against these enemies a grand and global alliance, North and South, East and West, that
can assure a more fruitful life for all mankind? Will you join me in this historic effort?
In the long history of the world, only a few generations have been granted the role of defending
freedom in its hour of maximum danger. I do not shrink from this responsibility; I welcome it I do not
believe that any of us would exchange places with any other people or any other generation. The
energy, the faith, the devotion which we bring to this endeavor will light our country and all who serve
it, and the glow from that fire can truly light the world.
And so, my fellow Americans, ask not what your country can do for you; ask what you can do for your
country.
My Fellow citizens of the world ask not what America will do for you, but what together we can do for
the freedom of man.
In the speech, Kennedy paints a picture of the United States as a nation that is
A. longing to return to the past
B. on the brink of world war
C. struggling to survive
D. the leading defender of freedom
Page 13 of 55
42. Which of the following will occur if a cold bottle of soda is left open on a kitchen counter?
A. The pressure that the soda exerts on the bottle will increase.
B. The temperature of the soda will decrease.
C. The amount of dissolved carbon dioxide gas will decrease.
D. The amount of dissolved carbon dioxide gas will remain the same.
43. In how many ways can you arrange three mathematics books (Algebra, Geometry, Trigonometry)
in order on a shelf?
A. 6
B. 8
C. 12
D. 24
E. none of these
44. What are the next four numbers in this sequence 8, 5, 4, 9,17, ___, ___, ___, ___?
A. 4, 3, 2, 1
B. 5, 4, 3, 2
C. 6, 3, 2, 0
D. 3, 2, 1, 0
E. none of these
45. A politician wants to get his message to 2/3 of the population of 48,000 in Bulacan. However, his
advertising campaign reaches only 3/4 of the number he intended. How many people does he actually
reach?
A. 16,000
B. 10,000
C. 24,000
D. 36,000
E. none of these
46. Aling antas ng tona ang lumilitaw sa bahaging may salungguhit ng pangungusap na nagdududa?
Page 14 of 55
Nagpuputol ng puno ang lalaki.
A. 1
B. 2
C. 4
D. 3
47. Which is NOT among the hazardous effects of water pollution to health?
A. The epidemic threat of hepatitis and dysentery
B. The increase incidence of liver cancer
C. The dumping of mercury in the sea causing blindness, brain damage, or death
D. The presence of certain bacteria in the digestive tract causing methemoglobinemia
48. Aling antas ng tono ang lumilitaw sa bahaging may salungguhit ng pangungusap na
nagsasalaysay?
Magbabasa ng mga gawain ang guro.
A. 1
B. 3
C. 2
D. 4
49. A tightly coiled spring demonstrates?
A. Steam energy
B. Kinetic energy
C. Potential energy
D. Chemical energy
50. Ano ang pinakaangkop na kahulugan nito?
Nagsasaya tayo ngayon sapagkat ang iyong namatay na kapatid ay muling nabuhay; ang nawawala
Page 15 of 55
ay muling nakita.
A. Ang pagbabalik ay dapat ipaghanda nang malaki.
B. Ang pagbabago ng kapatid ay dapat pahalagahan.
C. Dapat silang magsaya sa muli nilang pagsasama-sama
D. Ang pagsasama nila ay dahil sa muling pagbabalik ng kapatid.
51. The poem below is entitledSuburban Prophecywhich is written by Howard Nemerov.
On Saturday, the power-mowers' whine
Begins the morning. Over this neighbourhood
Rises the keening, petulant voice, begin
Green oily teeth to chatter and munch the cud.
Monsters, crawling the carpets of the world,
Still send from underground against your blades
The roots of things battalions green and curled
And tender, that will match your blades with blades
Till the revolted throats shall strangle on.
The tickle of their dead, till straws shall break
Crankshafts like camels, and the sun go down
On dinosaurs in swamps. A night attack
Follows and by the time the Sabbath dawns
All armored beasts are eaten by their lawns.
The imagery in the first stanza appeals to the reader's sense of
A. sight
B. touch
C. smell
D. hearing
52. Which part of the Allied action has been detailed in this passage?
After Saddam Hussein violated international agreements by sending Iraq troops to Kuwait and missiles
into other neighboring countries, the Allies responded with military action.
A. The Allied bases in Saudi Arabia
B. The ground was in the desert
C. The movement of Allied Troops with Iraq
D. The air campaign in the Baghdad area
Page 16 of 55
53. Which of the following words DOES NOT contain the voiceless Th?
A. Mouth
B. Breath
C. Teeth
D. Health
54. Which of the following is the BEST example of self-preservation?
A. mouse runs when it sees a cat.
B. A dog barks when it sees its owner.
C. A young man decides to quit smoking
D. A salmon swims back to the place of its birth to lay eggs.
55. Carl Sundburg wrote "Jazz Fantasia" which has for its first stanza:
Drum on your drums, batter on your banjos, sob on the long cool winding saxophones. Go to it, O
jazzmen.
Which words illustrate alliteration?
A. Batter and banjos
B. Long and cool
C. Sob and winding
D. To and it
56. Which of the following lines is a simile?
A. Holding wonder like a cup
B. Life has loneliness to sell
C. Eyes that love you, arms that hold
D. Buy it and never count the cost
57. What is the MOST likely reason for a desert plant to have a few or no leaves?
A. To increase photosynthesis
B. To decrease photosynthesis
Page 17 of 55
C. To increase transpiration
D. To decrease transpiration
58. Which are limited only to the sale of real property and stock transaction?
A. Business incomes
B. Capital gains
C. Employment incomes
D. Passive incomes
59. What is the mood of these lines?
Daylight, I must wait for the sunrise
I must think of a new life
And I mustn't give in.
When the dawn comes
tonight will be a memory, too
And a new day will begin.
A. Afraid
B. Sarcastic
C. Depressed
D. Hopeful
60. Below is an excerpt from John F. Kennedy's "Inaugural Address."
In your hands, my fellow citizens, more than mine, will rest the final success or failure of our course.
Since this country as founded, each generation of Americans has been summoned to give testimony to
its national loyalty. The graves of young Americans into answered the call to service surround the
globe.
Now the trumpet summons us again-not as a call to bear arms, though, arms we need, not as a call to
battle, though embattled we are; but a call to bear the burden of a long twilight struggle, year in and
year out, rejoicing in hope, patient in tribulation, a, struggle against the common enemies of man:
tyranny, poverty, disease, and war itself.
Can we forge against these enemies a grand and global alliance, North and South, East and West, that
can assure a more fruitful life for all mankind? Will you join me in this historic effort?
In the long history of the world, only a few generations have been granted the role of defending
freedom in its hour of maximum danger. I do not shrink from this responsibility; I welcome it I do not
believe that any of us would exchange places with any other people or any other generation. The
energy, the faith, the devotion which we bring to this endeavor will light our country and all who serve
it, and the glow from that fire can truly light the world.
And so, my fellow Americans, ask not what your country can do for you; ask what you can do for your
country.
My Fellow citizens of the world ask not what America will do for you, but what together we can do for
Page 18 of 55
the freedom of man.
The speech is characterized by all of the following stylistic devices EXCEPT
A. sad
B. uplifting
C. light-hearted
D. sarcastic
61. How many ways can a committee of 4 people be selected from a group of 7 people?
A. 35
B. 70
C. 140
D. 210
E. none of these
62. Which word contains the voiced Th?
A. Thank
B. These
C. Think
D. Thing
63. Which is the BEST WAY to write the underlined portion of this sentence?
However, if a fire extinguisher is handy, a quick-thinking person often can use them to put out a small
fire.
A. they
B. him
C. them
D. it
64. What indoor relative humidity range would probably be comfortable when the outside temperature
and humidity levels are extremely low?
A. 90 to 100%
B. 50 to 70%
Page 19 of 55
C. 20 to 30%
D. 30 to 40%
65. What do you predict will happen when you bring two bar magnets closer together?
A. They will repel each other.
B. They will create an alternating current
C. Nothing will happen
D. They will attract each other.
66. Kaninong tula hango ang sumusunod?
Ang hindi magmahal sa sariling wika mahigit sa hayop at malansang isda.
A. Jose Rizal
B. Emilio Jacinta
C. Apolinario Mabini
D. Graciano Lopez Jaena
67. Which word is read on a high note to describe the kind of day?
This is a cold day.
A. is
B. cold
C. day
D. this
68. The Jones family has had four children, all girls. The fifth child born is a boy. This change is the
result of
A. conception classes taken by the parents.
B. the timing of the fertility cycles.
C. the father's contribution of a "Y" chromosome.
D. the "law of averages" finally catching up.
69. Which one is the right to human dignity?
Page 20 of 55
A. Choose the goals and means of development
B. Share in scientific and technological advances of the world
C. Right to information
D. Sovereignty over our natural resources
70. Which is the BEST WAY to write the underlined portion of this sentence?
Researchers also speculate that some teachers might have given boys more computer time because
parents and teachers expected, boys to need computers for future careers.
A. will expect
B. expected
C. will have expected
D. expecting
71. Below is a biographical sketch.
Success often comes to those with humble beginnings, Elvis Aron Presley was born on January 8,
1935, in Tupelo, Mississippi. He first sang in a church choir and taught himself to play the guitar, but
he never learned to read music. By 1953, he had moved to Memphis, Tennessee, graduated from high
school, and enrolled in night school to become an electrician. That year, at Sun Records, Presley
recorded a personal record for his mother, a song that was heard by the company's president. As a
result of the president's recognition, Presley's first record,That's all Right, Mama,was out in 1954.
He toured the South, and in 1955 five of his records were released simultaneously. His first national
television appearance was that year on Jackie Gleason'sThe Stage Show,but Presley became known
for his appearance onThe Ed Sullivan Show,where the young singer gyrated as he sangrock n'
rollmusic. During the live television performance, Presley was photographed only from the waist up
because his motions were considered obscene.
Elvis the Pelvisbegan his film career in 1956 with LOVE ME TENDER and signed a long term film
contract. The movie critics were not always kind, but teenagers flocked to Presley's films. Within a few
short years, Presley had established a career that would span twenty-five years of ups and downs and
make him one of the most popular entertainers in history. Long after his untimely death at age 42,
Presley would be remembered asThe King of Rock n' Roll
The last sentence reveals that the author's attitude toward Presley is one of
A. indifference
B. admiration
C. disbelief
D. disgust
Page 21 of 55
72. What is the sum of all the two digit numbers which are divisible by 5?
A. 945
B. 950
C. 960
D. 1050
73. In an experiment, a vacuum is created when all air is removed from a tube. A coin and bits of
confetti are released in the vacuum at the same time. They fall at the same rate and reach the bottom
at the same time. The experiment proves that
I. in a vacuum, the rate of acceleration is the same for all objects regardless of weight.
II. outside a vacuum, air resistance is what makes different objects fall at different rates.
III. gravity has no effect at all on objects that fall in a vacuum.
A. I and II
B. I and III
C. I, II and III
D. II and III
74. Mary Rose, an 18-year-old was sexually abused by 3 teenagers from well-to-do families from
Makati. Despite pressures, she came out into the open to get justice. Which need did Mary Rose
satisfy?
A. Need for family unity
B. Need for civic responsibility
C. Need for universal solidarity
D. Need for personal integrity
75. Below is the poem written by Edgar Lee Masters in 1915:
What Happened to Tom Merritt?
At first I suspected something
She acted so calm and absent-minded.
And one day I heard the back door shut,
As I entered the front, and I saw him slink
Back of the smoke house into the lot,
And across the field.
And I meant to kill him on sight.
Page 22 of 55
But that day, walking near Fourth Bridge,
Without a stick or a stone at hand,
All of a sudden I saw him standing,
Scared to death, holding his rabbits,
And all l could say was,Don't, Don't, Don't,
As he aimed and fired at my heart.
Which of the following techniques is used in the poem?
A. Verse
B. Rhyme
C. Free verse
D. Personification
76. To pass her English Test, Lucille must get 75% of the items correct. Out of 80 questions, how
many must she correctly answer?
A. 55
B. 60
C. none of these
D. 65
E. 70
77. 4 1/5 + 3 2/7 = __________
A. 7 3/12
B. 7 3/35
C. 7 17/35
D. 7 1/35
E. none of these
78. The principle under which a thermostat operates is the same when
A. a gas expands to fill the container in which it is held.
B. a pendulum swings when it is set into motion.
C. a chemical reaction occurs when two substances combine.
D. the level of mercury rises or falls in a glass tube.
Page 23 of 55
79. Which is the BEST WAY to write the underlined portion of this sentence?
A person should keep in mind some basic safety rules when you are deciding whether or not to use a
fire extinguisher.
A. rules you decide
B. rules when you decided
C. rules you are deciding
D. rules when deciding
80. Which is the MOST important perceived need and problem of the Filipino family?
A. Unemployment or financial problem
B. Proneness to vices
C. Over protectiveness of children
D. Double standard on the roles of male and female
81. Which one BEST defines personal integrity?
A. The unity between ignorance and reality
B. The unity of man's deeds, words, thoughts and realities
C. The unity of man's social, political and physical aspects
D. The relationship between virtue and conduct
82. Which is the length of the hypotenuse of a right triangle with legs 5 inches and 12 inches?
A. 17in.
B. 13 in.
C. 11in.
D. 20 in.
E. none of these
83. Ang sumusunod ay isang talumpati na may pamagat na SA KABATAAN na sinulat ni Onofre
Pagsanghan.
Isa sa mga salitang napag-aralan natin sa wikang Pilipino ay ang salitang NABANSOT.
Kapang ang isang bagay daw ay dapat pang lumaki ngunit ito'y tumigil na sa paglaki, ang bagay na
ito raw ay NABANSOT. Marami raw uri ng pagkabansot ngunit ay pinakamalungkot na uri raw ay ang
pagkabansot ng isipan, ng puso at ng diwa.
Page 24 of 55
Ang panahon ng kabataan ay panahon ng paglaki, ngunit ang ating paglaki ay kailangang paglaki, at
pag-unlad ng ating buong katauhan, hindi lamang ng ating sukat at timbang. Kung ga-poste man ang
ating taas at ga-pison man ang ating bigat ngunit kung ang pag-iisip naman nati'y ga-kulisap lamang
kay pangit na kabansutan. Kung tumangkad man tayong tangkad-kawayan at bumilog man tayong
bilog-tapayan, ngunit kung tayo nama'y tulad nibondyingay di mapagkatiwalaan-anong laking
kakulangan. Kung magkakatawan tayong katawangTarzanat mapatalas ang ating isipang sintalas ng
kay Rizal, ngunit kung ang ating kalooban nama'y itim na duwende ng kasamaan-anong kapinsalaan
para sa kinabukasan.
Kinabukasan, kabataan, tayo raw ang pag-asa ng Inang Bayan. Tayo raw ang maghahatid sa kanya sa
langit ng kasaganaan at karangalan o hihils sa kanya sa putik ng kahirapan at kahihiyan. Ang panahon
ng pagkilos ay ngayon, hindi bukas, hindi sa isang taon. Araw-araw ay tumutuwid tayong palangit or
bumabaluktod tayong paputik. Tamang-tama ang sabi ng ating mga ninunong kung ano raw ang
kinamihasnan ay siyang pagkakatandaan. Huwag nating akalaing makapagpapabaya tayo ng ating
pag-aaral ngayon at sa araw ng bukas ay bigla tayong magiging mga dalubhasang magpapaunlad sa
bayan. Huwag nating akalaing makapaglulublob tayo ngayon sa kalaswaaan at kahalayan at sa
mahiwagang araw ng bukas sigla tayong magiging ulirang mga magulang.
Kabataan, tunay na pag-ibig sa bayan, ang tunay na nasyonalismo, ay wala sa tamis ng pangarap
wala rin sa pagpag ng dila. Ang tunay na pag-ibig ay nasa pawis ng gawa.
Bakit di dapat tumangkad tulad nibondying?
A. Di ito magpakakatiwalaan
B. Di totoo ito
C. May kakulangan ito
D. Magulo kasi ito
84. Lucy's husband has been a drug dependent. She wanted him rehabilitated to be economically
productive. Where will she commit her husband?
A. DARN
B. Bukang Liwayway Center
C. DARE
D. NFPI
85. Two days after Japan attacked Pearl Harbor, Roosevelt made the following statements:
In the past few years and most violently in the past few days, we have learned a terrible lesson. We
must begin the great task that is before us by abandoning once and for all the illusion that we can
never again isolate ourselves from the rest of humanity.
In the statement, Roosevelt is expressing the ideas of __________.
A. an internationalist
Page 25 of 55
B. an anti-imperialist
C. an imperialist
D. an isolationist
86. Which is the sum of the infinite progression 3/2, 1, 2/3, 4/9 . . . ?
A. 6½
B. 5½
C. 4½
D. 7½
E. none of these
87. How much larger is the supplement of a 57 degree angle than the complementof a 75 qegree
angle?
A. 108 degrees
B. 18 degrees
C. 105 degrees
D. none of these
E. 123 degrees
88. Ang sumusunod ay isang talumpati na may pamagat na SA KABATAAN na sinulat ni Onofre
Pagsanghan.
Isa sa mga salitang napag-aralan natin sa wikang Pilipino ay ang salitang NABANSOT.
Kapang ang isang bagay daw ay dapat pang lumaki ngunit ito'y tumigil na sa paglaki, ang bagay na
ito raw ay NABANSOT. Marami raw uri ng pagkabansot ngunit ay pinakamalungkot na uri raw ay ang
pagkabansot ng isipan, ng puso at ng diwa.
Ang panahon ng kabataan ay panahon ng paglaki, ngunit ang ating paglaki ay kailangang paglaki, at
pag-unlad ng ating buong katauhan, hindi lamang ng ating sukat at timbang. Kung ga-poste man ang
ating taas at ga-pison man ang ating bigat ngunit kung ang pag-iisip naman nati'y ga-kulisap lamang
kay pangit na kabansutan. Kung tumangkad man tayong tangkad-kawayan at bumilog man tayong
bilog-tapayan, ngunit kung tayo nama'y tulad nibondyingay di mapagkatiwalaan-anong laking
kakulangan. Kung magkakatawan tayong katawangTarzanat mapatalas ang ating isipang sintalas ng
kay Rizal, ngunit kung ang ating kalooban nama'y itim na duwende ng kasamaan-anong kapinsalaan
para sa kinabukasan.
Kinabukasan, kabataan, tayo raw ang pag-asa ng Inang Bayan. Tayo raw ang maghahatid sa kanya sa
langit ng kasaganaan at karangalan o hihils sa kanya sa putik ng kahirapan at kahihiyan. Ang panahon
ng pagkilos ay ngayon, hindi bukas, hindi sa isang taon. Araw-araw ay tumutuwid tayong palangit or
bumabaluktod tayong paputik. Tamang-tama ang sabi ng ating mga ninunong kung ano raw ang
kinamihasnan ay siyang pagkakatandaan. Huwag nating akalaing makapagpapabaya tayo ng ating
pag-aaral ngayon at sa araw ng bukas ay bigla tayong magiging mga dalubhasang magpapaunlad sa
Page 26 of 55
bayan. Huwag nating akalaing makapaglulublob tayo ngayon sa kalaswaaan at kahalayan at sa
mahiwagang araw ng bukas sigla tayong magiging ulirang mga magulang.
Kabataan, tunay na pag-ibig sa bayan, ang tunay na nasyonalismo, ay wala sa tamis ng pangarap
wala rin sa pagpag ng dila. Ang tunay na pag-ibig ay nasa pawis ng gawa.
Sa alin makikita ang tunay na NASYONALlSMO?
A. Diwa
B. Gawa
C. Sulat
D. Salita
89. Alin sa mga salita ang kasingkahulugan ng salitang may salungguhit?
Ang ama ni Anita ay kilalang bulanggugo sa kanilang lalawigan.
A. Laging ibinubulong
B. Laging handang gumasta
C. Laging handang makipag-away
D. Laging handang makipagtalo
90. What values are being given priority by juries in criminal cases?
A. The rights of the criminal over the strict interpretation of the law
B. The safety of the community over symphathy for the criminal
C. The needs of the criminals over the advice of the judge
D. The punishment of the criminal over the safety of the community
91. What correction should be made to this sentence?
Recently, educators exammined the effectiveness of computer instruction in schools.
A. Replace educators with educator's
B. Change the spelling of exammined to examined
C. Change schools to Schools
D. Replace computer with computer's
92. Si Dr. Jose Rizal ay sumulat ng aklat ng itinampok sa iba't ibang bansa.
Ang pangungusap ay nagagamit bilang __________?
Page 27 of 55
A. panuring
B. pamuno
C. tuwirang layon
D. paksa
93. A nation in which loess would likely be found today is
A. Iceland
B. United States
C. Japan
D. Ecuador
94. Which method of reproduction provides for the most variety of offspring?
A. Cloning
B. Sexual reproduction
C. Asexual reproduction
D. Cellular reproduction
95. Ano ang ipinahihiwatig ng salitang may salungguhit?
Matayog ang lipad ni Pepe kaya't bata pa siya'y nagsisikap na siya.
A. May kayabangan si Pepe.
B. Mataas ang pangarap ni Pepe.
C. Marunong si Pepe.
D. Ibig ni Pepeng maabot ang langit.
96. Below is an excerpt from John F. Kennedy's "Inaugural Address."
In your hands, my fellow citizens, more than mine, will rest the final success or failure of our course.
Since this country as founded, each generation of Americans has been summoned to give testimony to
its national loyalty. The graves of young Americans into answered the call to service surround the
globe.
Now the trumpet summons us again-not as a call to bear arms, though, arms we need, not as a call to
battle, though embattled we are; but a call to bear the burden of a long twilight struggle, year in and
year out, rejoicing in hope, patient in tribulation, a, struggle against the common enemies of man:
tyranny, poverty, disease, and war itself.
Can we forge against these enemies a grand and global alliance, North and South, East and West, that
Page 28 of 55
can assure a more fruitful life for all mankind? Will you join me in this historic effort?
In the long history of the world, only a few generations have been granted the role of defending
freedom in its hour of maximum danger. I do not shrink from this responsibility; I welcome it I do not
believe that any of us would exchange places with any other people or any other generation. The
energy, the faith, the devotion which we bring to this endeavor will light our country and all who serve
it, and the glow from that fire can truly light the world.
And so, my fellow Americans, ask not what your country can do for you; ask what you can do for your
country.
My Fellow citizens of the world ask not what America will do for you, but what together we can do for
the freedom of man.
One of the purposes of the speech is to motivate listeners to
A. serve their country
B. enlist in the armed forces
C. prepare themselves for battle
D. preserve the right to bear arms
97. Which is the BEST evidence that helium gas is lighter than air?
A. Helium has the lowest boiling point of all elements.
B. Helium atoms do not combine with other air atoms.
C. Helium-filled balloons rise in air.
D. By volume, helium makes up only 0.0005% of air.
98. Spouses Jose S. Luz and Celerina Luz filed a petition to adopt Gregorio Luz Ona, their nephew. The
spouses are childless and they reared Gregorio from his birth in 1971 until 1975 and they continued to
support him. Gregorio had to be left in the Philippines when the spouses went to the United States
where Jose is employed. The MSSD recommended the adoption to the court on the premise that
petitioners are in a better position to provide for the minor child than the natural parents who are
impoverished. What could be the possible decision of the court on the petition?
A. The court denied the petition because the spouses are already aliens
B. The petition for adoption was granted because the court finds that it is to the best
interest of the child.
C. The petition for adoption was denied because the spouses are non-residents of the Philippines.
D. The court denied the petition on the premise that the trial custody requiredby PD 603 cannot
be effected for spouses are non-residents
99. As a representative of the Urban Poor Commission of the Association of Religious Superiors (ARS),
which action will you most likely take to resolve the long-term roots of structural inequalities-
proliferation of child labor and child prostitution?
Page 29 of 55
A. Raise views of human rights abuse.
B. Organize regular programs for information and discussion of human rights.
C. Conduct skills training.
D. Raise questions over the government's commitment to rebuild human rights
100. The main message of the Moral Recovery Program launched by Leticia Ramos Shahani starts with
the __________.
A. world
B. family
C. self
D. nation
101. The Miranda Family purchased a 250-pound side of beef and had it packaged. They paid
Php365.00 for the side of beef. During the packaging, 75 pounds of beef were discarded as waste.
What was the cost per pound for the packaged beef?
A. Php2.08 per pound
B. none of these
C. Php2.50 per pound
D. Php2.06 per pound
E. Php2.30 per pound
102. Which has become a prototype of other schemes that defeated the real and true purpose of the
CARL?
A. The conversion of farmlands to industrial complexes
B. The stock option scheme of Hacienda Luisita
C. The conversion from agricultural to subdivision
D. The voluntary offer to sell
103. A carpenter wanted three pieces of wood each 1 1/2 feet long. If he planned to cut them from a
6-foot piece of wood, how much of the piece would be left?
A. 4 1/2 ft
B. 1 1/2 ft
C. 4 1/2 ft
Page 30 of 55
D. 3 ft
E. none of these
104. Which pitch is used for the word STRANGE?
What a strange story!
A. 3
B. 2
C. 4
D. 1
105. Which of the following methods can all diabetics control their condition and avoid heart disease
and blindness?
I. Regulating their intake of glucose
II. Increasing the levels of insulin in the body by taking insulin injections
III. Maintaining a reasonable exercise regimen to keep weight down.
A. I
B. II
C. I and II
D. I and III
106. How many gallons of water will fill a fish tank that is 18 inches by 12 inches by 48 inches? (There
are 231 cubic inches per gallon). Round your answer to the nearest gallon.
A. 45 gallons
B. 40 gallons
C. none ofthese
D. 47 gallons
E. 38 gallons
107. Which is the BEST WAY to write the underlined portion of this sentence?
The hosts also benefit from running such a business because they can stay at home, make
money, and meeting a variety of people.
A. get to meet
Page 31 of 55
B. to meet
C. and meet
D. and be meeting
108. New peonies, perennial plants that produce showy flowers can be propagated from the parent
plant by dividing corns that grow underground. This reproductive form resembles a
A. bulb
B. runner
C. seed
D. bud
109. Alin sa mga sumusunod ang mensahe ng epiko ng IIokano na BIAG NI LAM-ANG?
A. Pinatutunayan ng epiko ang yaman ng Ilokano sa lahat ng bagay.
B. Kailangan paniniwalaan ang ukol sa bisa ng mga anting-anting dahil sa mga pangyayaring
nagpapatunay dito.
C. Dito nagpapatunay na walang kamatayan.
D. Masasalat ang mga katutubong ugali at mga tradisyong dapat pagyamanin at
panatilihin upang pakinabangan ng kabataan.
110. The Soviet Union's attempt to establish a missile base in Cuba is interpreted as a direct violation
of
A. the Truman DoCtrine
B. the Monroe Doctrine
C. the Strategic Arms Limitations Talks (SALT)
D. the Declaration of Independence
111. What correction should be made to this sentence?
Most fire-related death's result from household fires; yet many people do not have fire extinguishers
in their homes.
A. Remove the comma after fires
B. Change result to results
C. Change have to has
D. Replace death's with deaths
Page 32 of 55
112. Employees at Shaira's Musicmart get a 20% discount on all purchases. If Theresa buys three
tapes at Php7.49 each, how much will she have to pay after her employee discount?
A. Php16.98
B. Php19.98
C. Php18.98
D. Php17.98
E. none of these
113. Which family obligation is especially valued?
A. Supplying groceries to relatives in remote barrios
B. Providing health assistance to relatives living in the same locale
C. Sending to college relatives in remote barrios
D. Keeping immediate family members out of trouble
114. Which word in the passage does NOT require a change in pitch to show confidence?
I am the master of my fate, I am the captain of my soul.
A. am
B. captain
C. master
D. fate
115. Mang Tacio has been unemployed for quite sometime due to his negative attitudes toward work.
Which program of the Department of Social welfare and Services will help him?
A. Income in Kind Program
B. Anti-Medicancy program
C. HRD Program
D. Social Insurance Program
116. Scientist also find that other stalky vegetables such as carrots also help lower blood pressure.
This statements is BEST classified as
A. experiment
B. nonessential fact
Page 33 of 55
C. finding
D. prediction
117. Alin uri ng parirala ang may salungguhit sa pangungusap?
Utang sa kanyang sipag at sikap sa paggawa ang kanilang maalwang pamumuhay.
A. Pangngalan
B. Pangngalang-diwa
C. Pawatas
D. Pang-ukol
118. In which kingdom should MOLD be classified?
A. Protista
B. Plantae
C. Fungi
D. Animalia
119. What is meant by TWO PINS in this sentence?
For two pins I could have hit him on the nose.
A. A second course of action
B. Without much persuading
C. The second chance
D. Have a second alternative choice
120. Which is the BEST WAY to write the underlined portion of this sentence?
There is smoke detectors in many homes to warn residents of a fire, but fire extinguishers can actually
help people fight fires.
A. They're is
B. Their is
C. Their are
D. There are
Page 34 of 55
121. Alin sa mga sumusunod ang aral na ibinigay ng ANG ALAMAT NI MARIANG MAKILlNG na
ikinuwento ni Jose Rizal?
A. Pagyamanin at pangalagaan ang ating bayan at lahi pagka’t hiyas at yaman natin ito.
B. Pag-ibig ang makapagbabago sa mundong ito.
C. Kabanalan ang magpatawad at tulungan ang isang nagkasala.
D. Dahil sa pagmamalabis at pagsasamantala, maraming biyaya ang sa kanya'y
nawawala.
122. Which one should be TRUE if Earth's rotation axis did not tilt?
I. Days and nights would be the same length everywhere on Earth.
II. There would be no hours of darkness on points along the equator.
III. Earth would have no seasons.
IV. Each part of the Earth would have the same daily temperature pattern.
A. II and IV
B. III and IV
C. IV and I
D. I and III
123. The San Pascual Credit Cooperative of Quezon City wishes to appy for a loan of five million pesos
from one of the financial institution, EXCEPT
A. Development Bank of the Philippines
B. Central Bank of the Philippines
C. Philippine National Bank
D. Land Bank of the Philippines
124. What is meant by SOFT OPTION in these sentences?
Rebecca realized that if she stayed in her present job it would mean competing with an envious rival.
Leaving the company would probably be a soft option.
A. An action that is difficult to take
B. An action that is not agreeable
C. An action that is easier.
D. An action that is weakly funded
Page 35 of 55
125. Which of the following foreign policy actions today is a direct result of early American imperialist
policy?
A. Stationing of American troops in West Germany
B. America's military support of Israel in the Middle East
C. America's patrolling of waters off the Libyan coast
D. America's establishment of military bases in the Philippines
126. Anong tayutay ang tinutukoy nito?
Durog ang katawang bumagsak sa semento si Miguel.
A. Pagtutulad
B. Pagbibigay katauhan
C. Pagmamalabis
D. Pagwawangis
127. Mr. Garcia owns a 10½ hectare tract of land. He plans to subdivide this tract into ¼ hectare lots.
He must first set aside 1/16 of the total land for roads. How many lots will this tract yield?
A. 30
B. 35
C. 42
D. 45
E. none of these
128. The following are legitimate children, EXCEPT
A. those born by artificial insemination.
B. those legitimated.
C. those born during a valid marriage of parents.
D. those born out of a valid marriage of the parents.
129. The stanza below is taken from "Barter" by Sara Teasdale.
Life has loneliness to sell,
Music like a curve of gold,
Scent of pine trees in the rain,
Eyes that love you, arms that hold,
Page 36 of 55
And for your spirit's still delight,
Holy thoughts that star the night.
To what does Teasdale compare music?
A. The scent of pine trees
B. A curve of gold
C. Eyes that love
D. The rain
130. Which is the BEST WAY to write the underlined portion of this sentence?
The studies revealing that, for various reasons, girls spent less time working with computers' than
boys.
A. Revealing studies
B. Studies revealed
C. Studies' revelations
D. Studies will reveal
131. Sabihin ang aspekto ng pandiwa sa pangungusap na ito.
Mag-aral sa bahay ng mga araling ukol sa halaman.
A. Pawatas
B. Kontemplatibo
C. Imperpektibo
D. Perpektibo
132. 10 - 3 2/17 = __________
A. 7 2/1 7
B. 6 2/1 7
C. 6 1/17
D. 6 15/17
E. none of these
133. Below is a biographical sketch.
Success often comes to those with humble beginnings, Elvis Aron Presley was born on January 8,
Page 37 of 55
1935, in Tupelo, Mississippi. He first sang in a church choir and taught himself to play the guitar, but
he never learned to read music. By 1953, he had moved to Memphis, Tennessee, graduated from high
school, and enrolled in night school to become an electrician. That year, at Sun Records, Presley
recorded a personal record for his mother, a song that was heard by the company's president. As a
result of the president's recognition, Presley's first record,That's all Right, Mama,was out in 1954.
He toured the South, and in 1955 five of his records were released simultaneously. His first national
television appearance was that year on Jackie Gleason'sThe Stage Show,but Presley became known
for his appearance onThe Ed Sullivan Show,where the young singer gyrated as he sangrock n'
rollmusic. During the live television performance, Presley was photographed only from the waist up
because his motions were considered obscene.
Elvis the Pelvisbegan his film career in 1956 with LOVE ME TENDER and signed a long term film
contract. The movie critics were not always kind, but teenagers flocked to Presley's films. Within a few
short years, Presley had established a career that would span twenty-five years of ups and downs and
make him one of the most popular entertainers in history. Long after his untimely death at age 42,
Presley would be remembered asThe King of Rock n' Roll
The statement thatsuccess often comes to those with humble beginningswould apply best to which of
the following figures?
A. Ramon Magsaysay
B. Gloria M. Arroyo
C. Corazon C. Aquino
D. Joseph E. Estrada
134. At which time during the year does the ozone level present a particular health threat in urban
areas for people with respiratory problems?
A. Spring
B. Fall
C. Summer
D. Winter
135. Why was San Andres Cooperative Association of Paco, Manila not exempted from taxation?
A. It accumulated reserves and undivided net savings of Php8,000,000.00
B. It accumulated reserves and undivided net savings of Php10,000.00
C. It accumulated reserves and undivided net savings Php9,000,000.00
D. It accumulated reserves and undivided net savings of Php11,000.00
136. If a farmer would want assistance like pricing, guarantee for all agricultural produce or
cooperative management training, where would he go?
Page 38 of 55
A. Support Services of the Department of Agrarian Reform
B. Special Agrarian Court under the Regional Trial court
C. DAR Adjudication Board
D. Land Bank
137. Which of the following represent, ethnocentric behavior?
A. A tourist who lectures his foreign hosts on the "uncivilized" nature of their marriage
customs
B. A student who tutors an immigrant in English
C. A Hispanic community group demands that public aid forms be published in English and
Spanish
D. A peace Corps volunteer who helps dig wells in Central Africa
138. Which word is pronounced with a /z/ at the end?
A. Maps
B. Jakes
C. Laughs
D. Buys
139. Below is the poem written by Edgar Lee Masters in 1915:
What Happened to Tom Merritt?
At first I suspected something
She acted so calm and absent-minded.
And one day I heard the back door shut,
As I entered the front, and I saw him slink
Back of the smoke house into the lot,
And across the field.
And I meant to kill him on sight.
But that day, walking near Fourth Bridge,
Without a stick or a stone at hand,
All of a sudden I saw him standing,
Scared to death, holding his rabbits,
And all l could say was,Don't, Don't, Don't,
As he aimed and fired at my heart.
The poet introduces the poem with lines 1-3 to show us that Tom Merritt
A. had suspected that his wife was seeing another man
Page 39 of 55
B. was sure that his wife was ill
C. was a very suspicious person
D. was sure that his wife still loved him
140. Which of the following procedures used by a farmer is NOT related directly to preventing erosion?
A. Contour plowing around a hill
B. Planting more seeds than are necessary to yield a bountiful crop.
C. Planting grass in gullies to act as a filter
D. Planting crops in alternate rows (strip farming)
141. Which is the BEST WAY to write the underlined portion of this sentence?
However, their VCR kept them from missing their favorite primetime shows.
A. keepes
B. keeps
C. had kept
D. keeped
142. Below is a biographical sketch.
Success often comes to those with humble beginnings, Elvis Aron Presley was born on January 8,
1935, in Tupelo, Mississippi. He first sang in a church choir and taught himself to play the guitar, but
he never learned to read music. By 1953, he had moved to Memphis, Tennessee, graduated from high
school, and enrolled in night school to become an electrician. That year, at Sun Records, Presley
recorded a personal record for his mother, a song that was heard by the company's president. As a
result of the president's recognition, Presley's first record,That's all Right, Mama,was out in 1954.
He toured the South, and in 1955 five of his records were released simultaneously. His first national
television appearance was that year on Jackie Gleason'sThe Stage Show,but Presley became known
for his appearance onThe Ed Sullivan Show,where the young singer gyrated as he sangrock n'
rollmusic. During the live television performance, Presley was photographed only from the waist up
because his motions were considered obscene.
Elvis the Pelvisbegan his film career in 1956 with LOVE ME TENDER and signed a long term film
contract. The movie critics were not always kind, but teenagers flocked to Presley's films. Within a few
short years, Presley had established a career that would span twenty-five years of ups and downs and
make him one of the most popular entertainers in history. Long after his untimely death at age 42,
Presley would be remembered asThe King of Rock n' Roll
The author uses the phrase ups and downs to refer to Presley's
Page 40 of 55
A. gyrations as he performed
B. increasing and decreasing finances
C. successes and disappointments in his career
D. use of drugs,uppers and downers
143. In an experiment, a drop of blue ink is placed on the surface of a glass of water. In a few
minutes, the drop of ink is dispersed throughout the water, turning it light blue. The result of the
experiment proves that
A. molecules of ink and molecules of water are in constant motion.
B. heat causes the ink to disperse.
C. a new compound is formed by the combination of ink and water.
D. ink molecules have less density than water molecule.
144. An elderly woman suffered a stroke, a restriction of blood flow to the brain. If the stroke caused
to the right side of her body to become temporarily paralyzed, she most likely experienced a
decreased blood flow to
A. the left side of her body.
B. the front of her brain.
C. the left side of her brain.
D. the right side of her brain.
145. What is the function of DIFFUSION in the human body?
A. Regulates blood flow
B. Plays an insignificant role in the body's functioning
C. Allows an even distribution of substances throughout all cells of the body
D. Comes into play in times of extreme illness
146. Each of the following objects is designed to employ the buoyancy principle EXCEPT a
A. life preserver
B. kite
C. submarine
D. canoe
Page 41 of 55
147. It was also discovered that the chemical 3m butylphthalide can lower the blood pressure of rats.
This statement is classified as
A. prediction
B. experiment
C. finding
D. nonessential fact
148. Lines 11 and 12 are taken from the poem "maggie and milly and molly and may."
For whatever we lose (like a you or a me)
it's always ourselves we find in the sea
Which of the following ideas is the author expressing?
A. The sea is a source of life and death.
B. The sea represents all of our moods.
C. The sea is the best place for a person to reflect about life.
D. The sea and its surroundings can give people a fresh view on life.
149. Which refers to FUNNEL EFFECT?
A. The belief that every criminal gets caught and is punished.
B. The belief that crime is under control in the United States.
C. The idea that only a very few suspects arrested for committing a crime are actually
punished.
D. The idea that all crimes are put into the same criminal justice system.
150. Alin ang naayong pamagat sa tanagang sinulat ni Jose Villa Panganiban?
Ano man sa daigdig,
Maaring magamit,
Ano mang masaisip;
Di sukat maiipit.
A. Pagkainip
B. Paraya
C. Pag-asa
Page 42 of 55
D. Pagbibigay
151. The undeclared war in Korea most closely resembled the situation of
A. The Spanish-American War
B. The Vietnam War
C. World War I
D. World War II
152. Alin ang di-karaniwang anyo ng pandiwang HINTAY KA?
A. Tay
B. Tayka
C. Intay
D. Teka
153. Below is the poem written by Edgar Lee Masters in 1915:
What Happened to Tom Merritt?
At first I suspected something
She acted so calm and absent-minded.
And one day I heard the back door shut,
As I entered the front, and I saw him slink
Back of the smoke house into the lot,
And across the field.
And I meant to kill him on sight.
But that day, walking near Fourth Bridge,
Without a stick or a stone at hand,
All of a sudden I saw him standing,
Scared to death, holding his rabbits,
And all l could say was,Don't, Don't, Don't,
As he aimed and fired at my heart.
Who is the speaker of this poem?
A. Tom Merritt
B. Merritt's wife
Page 43 of 55
C. God
D. The sheriff
154. A car dealer is offering a rebate of Php7,500.00 on any new-car purchase. If the purchase price
of a car is Php200,000.00 more than it was last year, what is the rate of discount offered by the
rebate?
A. 10%
B. not enough information is given
C. 7.5%
D. 13.3%
E. 14.2%
155. Which of the following internal forces interrupt the external forces of erosion?
I. Forces that cause volcanoes
II. Forces that cause ocean trenchers
III. Forces that create mountains
A. I, II and III
B. I and III
C. II and III
D. I and II
156. Of the following, which is imposed a final tax of ten percent (10%)?
A. PCSO and lotto winnings
B. Books, literary works and musical compositions
C. Currency bank deposit
D. Royalties
157. What is meant by the expression TO GET BLOOD OUT OF A STONE in these sentences?
Geraldo has owed me fifty thousand pesos for over a year now. I’ve asked him for it on several
occasions, but it’s like trying to get blood out of a stone.
A. Something is impossible.
B. Someone refuses to cooperate.
Page 44 of 55
C. Someone is willing to give what is asked.
D. Someone wants revenge.
158. Four mangoes cost P29.00. At that price what will 2½ dozen mangoes cost?
A. Php217.50
B. Php188.50
C. Php348.50
D. Php870.00
E. none of these
159. Which kingdom should STREPTOCOCCUS be classified?
A. Protista
B. Plantae
C. Fungi
D. Monera
160. Which of the following should you expect to be true about the rate of cellular respiration for a
group of students who are the same age, height, and weight?
A. Athletes would tend to have higher rates of cellular respiration than non-athletes.
B. Africans would have a higher rate of cellular respiration than Asians.
C. Boys would have a higher rate of cellular respiration than girls.
D. Non-athletes would have higher rates of cellular respiration than athletes.
161. Without the process of meiosis, we can infer that offspring from sexual reproduction would
A. have a high degree of genetic variety.
B. have twice the assigned number of chromosomes.
C. be identical.
D. have a number of mutations.
162. You buy a new refrigerator for Php12,800.00 and make a down payment of Php2,500.00. If you
finance the remainder at 8% annually for three years, how much will you actually pay for the
refrigerator?
Page 45 of 55
A. Php12,190.00
B. Php10,300.00
C. none of these
D. Php12,772.00
E. Php15,272.00
163. Alin sa mga sumusunod ang aral na ibinibigay ng epikong Muslim na INDARAPATRA AT
SULAYMAN?
A. Pagmamahal
B. Katapangan
C. Katapatan,
D. Pagtanaw ng utang-na-loob
164. Which of the following facts support the Big Bang theory's explanation of the creation of the
universe?
A. The universe does not expand nor contract.
B. The universe seldoms expand.
C. The universe will have background radiation.
D. The universe has no beginning nor end.
165. Which are the next three terms in the progression 1/125, -1/25, 1/5 . . . 7 terms?
A. -2, 6, -26
B. -3, 7, -27
C. -4, 8, -28
D. -1, 5, -25
E. none of these
166. What correction should be made to this sentence?
First-borns often pattern their behavior after they're parents and other adults.
A. Replace their with they're
B. Change pattern to patterned
Page 46 of 55
C. Replace they're to their
D. No correction is necessary
167. The poem below is entitledSuburban Prophecywhich is written by Howard Nemerov.
On Saturday, the power-mowers' whine
Begins the morning. Over this neighbourhood
Rises the keening, petulant voice, begin
Green oily teeth to chatter and munch the cud.
Monsters, crawling the carpets of the world,
Still send from underground against your blades
The roots of things battalions green and curled
And tender, that will match your blades with blades
Till the revolted throats shall strangle on.
The tickle of their dead, till straws shall break
Crankshafts like camels, and the sun go down
On dinosaurs in swamps. A night attack
Follows and by the time the Sabbath dawns
All armored beasts are eaten by their lawns.
To what does the phraseyour bladesin line 8 refer?
A. Lawnmowers
B. Roots
C. Carpets
D. Monsters
168. How much topsoil is needed to cover a garden 25 feet by 40 feet to a depth of 6 inches?
A. 480 ft3
B. 440 ft3
C. 500 ft3
D. 460 ft3
E. none of these
169. Which word contains the /ae/ sound?
A. Carriage
B. Cabin
Page 47 of 55
C. Caste
D. Can
170. Which will solve poverty caused by capitalism?
A. Fascism
B. Communism
C. Empirism
D. Socialism
171. Which one is the human right to dignity?
A. Political independence
B. Honor and reputation
C. Form associations
D. Social and economic reforms
172. What correction should be made to this sentence?
Buying a fire extinguisher knowing how to use it, and placing it in a location familiar to all family
members can help protect families against fire.
A. Insert a comma after extinguisher
B. Change the spelling of families to familys
C. Change placing to place
D. Insert a comma after help
173. Which verb in the sentence is pronounced with the ending as /d/?
They laughed and joked as they walked and played.
A. Joked
B. Laughed
C. Walked
D. Played
174. What correction should be made to this sentence?
Page 48 of 55
Most State tourism departments and some travel agencies have bed and breakfast listings.
A. Insert a comma after agencies
B. Change have to has
C. Change tourism to Tourism
D. Change State to state
175. Which theory was asserted by the Pan-German belief in the superiority of the Aryan race and
that the strength of the German culture came from a strong, healthy and rustic lineage?
A. DependencyTheory
B. Culture of Poverty Theory
C. Social Darwinist Theory
D. Theory of Capitalism
176. The hypotenuse of a right triangle is 25 feet. If one leg is 24 feet, what is the length of the other
leg?
A. 5 ft
B. 5 ft
C. 20 ft
D. 7 ft
E. none of these
177. Where would you commit a drug dependent for him to achieve a natural, tensionless, and
anxiety-free state?
A. NFPI
B. DARN
C. DARE
D. Bukang Liwayway Center
178. Which is the equivalent common fraction of the repeating decimal 3.242424...?
A. 107/33
B. 110/33
C. 109/33
Page 49 of 55
D. 108/33
E. none of these
179. What is meant by AT SIXES AND SEVENS in this sentence?
We moved into the house last week, but I'm afraid everything, is still at sixes and sevens.
A. The things have not been shipped.
B. In a state of confusion
C. In an orderly manner
D. The boxes are still intact
180. What is suggested in the opening line?
June 13, 1986 - they came from all over America - 200,000 heroes strong, with their families.
A. The writer holds great admiration for the veterans
B. The writer was a veteran of the war
C. The writer is opposed to the Vietnam War
D. The writer is a flag-waving patriot
181. Which law of force and motion explains this occurrence, when a rocket is propelled upward by the
powerful downward discharge of exhaust gases?
A. Universal Gravitation
B. Action and Reaction
C. Applied Force
D. Inertia
182. Tides, caused by the moon's gravity, create a frictional force that is gradually slowing down
Earth's rotation speed. One million years from now, scientist may discover that compared to today,
Earth's
A. day is shorter
B. year is longer
C. day is longer
D. year is shorter
Page 50 of 55
183. Which area of the brain controls feeling on the left side of a person's face?
A. The left occipital lobe
B. The left parietal lobe
C. The right parietal lobe
D. The right occipital lobe
184. Which location should have most nearly twelve hours of daylight and twelve hours of darkness
during December? A town that is located
A. halfway between the Equator and the South Pole.
B. close to the Equator.
C. close to the North Pole.
D. close to the South Pole.
185. During a recent shopping spree, Tomas and Nena bought some new accessories for their
apartment. Nena chose a crocheted throw pillow at Php24.95, and Tomas purchased a rural landscape
painting for Php135.00. How much did they actually spend if they paid 7% sales tax on their
purchases?
A. Php171.15
B. Php139.25
C. none of these
D. Php159.95
E. Php148.75
186. Dadalaw sa mga paaralan si Dr. Filemon S. Salas, ang tagapamanihala ng mga paaralang
lungsod, sa lungsod ng Pasay.
Ang pangungusap ay nagagamit bilang
A. panuring
B. paksa
Page 51 of 55
C. tuwirang layon
D. pamuno
187. Ang sumusunod ay isang talumpati na may pamagat na SA KABATAAN na sinulat ni Onofre
Pagsanghan.
Isa sa mga salitang napag-aralan natin sa wikang Pilipino ay ang salitang NABANSOT.
Kapang ang isang bagay daw ay dapat pang lumaki ngunit ito'y tumigil na sa paglaki, ang bagay na
ito raw ay NABANSOT. Marami raw uri ng pagkabansot ngunit ay pinakamalungkot na uri raw ay ang
pagkabansot ng isipan, ng puso at ng diwa.
Ang panahon ng kabataan ay panahon ng paglaki, ngunit ang ating paglaki ay kailangang paglaki, at
pag-unlad ng ating buong katauhan, hindi lamang ng ating sukat at timbang. Kung ga-poste man ang
ating taas at ga-pison man ang ating bigat ngunit kung ang pag-iisip naman nati'y ga-kulisap lamang
kay pangit na kabansutan. Kung tumangkad man tayong tangkad-kawayan at bumilog man tayong
bilog-tapayan, ngunit kung tayo nama'y tulad nibondyingay di mapagkatiwalaan-anong laking
kakulangan. Kung magkakatawan tayong katawangTarzanat mapatalas ang ating isipang sintalas ng
kay Rizal, ngunit kung ang ating kalooban nama'y itim na duwende ng kasamaan-anong kapinsalaan
para sa kinabukasan.
Kinabukasan, kabataan, tayo raw ang pag-asa ng Inang Bayan. Tayo raw ang maghahatid sa kanya sa
langit ng kasaganaan at karangalan o hihils sa kanya sa putik ng kahirapan at kahihiyan. Ang panahon
ng pagkilos ay ngayon, hindi bukas, hindi sa isang taon. Araw-araw ay tumutuwid tayong palangit or
bumabaluktod tayong paputik. Tamang-tama ang sabi ng ating mga ninunong kung ano raw ang
kinamihasnan ay siyang pagkakatandaan. Huwag nating akalaing makapagpapabaya tayo ng ating
pag-aaral ngayon at sa araw ng bukas ay bigla tayong magiging mga dalubhasang magpapaunlad sa
bayan. Huwag nating akalaing makapaglulublob tayo ngayon sa kalaswaaan at kahalayan at sa
mahiwagang araw ng bukas sigla tayong magiging ulirang mga magulang.
Kabataan, tunay na pag-ibig sa bayan, ang tunay na nasyonalismo, ay wala sa tamis ng pangarap
wala rin sa pagpag ng dila. Ang tunay na pag-ibig ay nasa pawis ng gawa.
Alin salita ang paulit-ulit na binabanggit ni Onofre Pagsanghan?
A. Nabansot
B. Bayan
C. Kabataan
D. Kung
188. In the progression 18, -12, 8 . . . which term is 512/729?
A. the 8th
B. the 6th
C. the 9th
D. the 7th
E. none of these
Page 52 of 55
189. Why does a bullet when discharged into the air eventually fall to the ground? This is due to the
Law of
A. Universal Gravitation
B. Applied Force
C. Inertia
D. Action and Reaction
190. Kilalanin ang uri ng pariralang may salungguhit.
Ang pangangalaga sa mga likas na yaman ay tungkulin nating lahat.
A. Pangngalan
B. Pangngalang-diwa
C. Pang-ukol
D. Pawatas
191. If P75.00 is shared among three children in the ratio of 3:7:15. The size of the smaller share is
A. Php9.00
B. Php15.00
C. Php25.00
D. Php35.00
E. none of these
192. To gain the attention of the audience, the trick is __________?
A. start low, speak hurriedly
B. start high, speak loudly
C. start high, speak rapidly
D. start low, speak slowly
193. Which is the BEST WAY to write the underlined portion of this sentence?
Recently psychologists have been researching birth order, their research suggests that personality and
intelligence are based partly on where a child ranks in the family.
A. order. Their
Page 53 of 55
B. order and their
C. order, or their
D. order their
194. Anong aral ang ibinibigay ng sumusunod na salawikain?
Ang taong napapailalim, ay naipapaibabaw rin.
A. Maaring ngayon ay hirap pagdating ng bukas ay may ginhawa rin
B. Tiyak ang pag-unlad kapag nauna ang hirap
C. Kung ano ang ibig natin ay mangyayari
D. Magtiis kung dumarating ang hirap
195. Carolina Diaz filed a petition for habeas corpus against Mr. and Mrs. Ramon Alde to recover
custody of Lina Diaz Tan aliasGracia Alde,the natural daughter of Carolina Diaz, who was a hostess.
What could be the possible action of the court on the petition filed by Carolina Diaz?
A. Her petition would be granted because she now works as a clerk in a prestigious office.
B. Her petition would be denied because she was a former hostess.
C. Her petition would be denied because when Gracia was given to the Aldes it was tantamount
to abandonment of the child, resulting to termination of parental authority.
D. Her petition would be granted because she is the natural mother.
196. Which is the main goal of drug abuse education?
A. Arrest
B. Prevention
C. Control
D. Rehabilitation
197. Why did the register of deeds charge Lucio Cruz registration fee the instrument relative to his
loan?
A. His loan was Php30,000.00
B. His loan was Php60,000.00
Page 54 of 55
C. His loan was Php50,000.00
D. His loan was Php40,000.00
198. The poem below is entitledSuburban Prophecywhich is written by Howard Nemerov.
On Saturday, the power-mowers' whine
Begins the morning. Over this neighbourhood
Rises the keening, petulant voice, begin
Green oily teeth to chatter and munch the cud.
Monsters, crawling the carpets of the world,
Still send from underground against your blades
The roots of things battalions green and curled
And tender, that will match your blades with blades
Till the revolted throats shall strangle on.
The tickle of their dead, till straws shall break
Crankshafts like camels, and the sun go down
On dinosaurs in swamps. A night attack
Follows and by the time the Sabbath dawns
All armored beasts are eaten by their lawns.
The poet's use of words such as whine, voice, teeth, chatter and munch is to suggest that the power-
mowers are
A. very powerful
B. alive
C. like cows
D. green
199. Which explains the reason why there are continuous and increasing human rights violations?
A. The United Nation's General Assembly approved only resolutions on human rights
and the basic freedoms which are not binding.
B. The solutions used are ineffective.
C. The United Nations as an international body is rather slow in the exercise of its powers.
D. The United Nations uses a single solution on all forms of human rights violations.
200. What is meant by LAST DITCH in this sentence?
The aged bishop was prepared to fight to the last ditch to defend his good name.
A. One's last courage
Page 55 of 55
B. One's last strategy
C. One's last hope
D. One's last defense

Weitere ähnliche Inhalte

Was ist angesagt?

900 questions gen ed
900 questions gen ed900 questions gen ed
900 questions gen edchinnex23
 
Assessment and evaluation of learning part 3
Assessment and evaluation of learning part 3Assessment and evaluation of learning part 3
Assessment and evaluation of learning part 3chinnex23
 
FLCT Chapter 4 Module.docx
FLCT Chapter 4 Module.docxFLCT Chapter 4 Module.docx
FLCT Chapter 4 Module.docxMarilouOTamayo
 
Mga Likas na Yaman ng mga Lalawigan sa Rehiyon
Mga Likas na Yaman ng mga Lalawigan sa RehiyonMga Likas na Yaman ng mga Lalawigan sa Rehiyon
Mga Likas na Yaman ng mga Lalawigan sa RehiyonRitchenMadura
 
FS 1 Episode 3
FS 1 Episode 3FS 1 Episode 3
FS 1 Episode 3Yuna Lesca
 
Earth sciencetrivia
Earth sciencetriviaEarth sciencetrivia
Earth sciencetriviaErlierose
 
Kpup re examined final2
Kpup re examined final2Kpup re examined final2
Kpup re examined final2JOVINER LACTAM
 
Yamang Tubig sa Pilipinas
Yamang Tubig sa PilipinasYamang Tubig sa Pilipinas
Yamang Tubig sa PilipinasPrincess Sarah
 
Child and adolescent development part 2
Child and adolescent development part 2Child and adolescent development part 2
Child and adolescent development part 2chinnex23
 
What is teaching internship?
What is teaching internship?What is teaching internship?
What is teaching internship?Jurix Cabuyao
 
254457273-LET-Review-Assessment-of-Learning-Test-Items.pptx
254457273-LET-Review-Assessment-of-Learning-Test-Items.pptx254457273-LET-Review-Assessment-of-Learning-Test-Items.pptx
254457273-LET-Review-Assessment-of-Learning-Test-Items.pptxItsssClarizza
 
Principles and strategies of teaching part 3
Principles and strategies of teaching part 3Principles and strategies of teaching part 3
Principles and strategies of teaching part 3chinnex23
 
Licensure examination for teachers1
Licensure examination for teachers1Licensure examination for teachers1
Licensure examination for teachers1Mina Olario
 
Principles and strategies of teaching part 2
Principles and strategies of teaching part 2Principles and strategies of teaching part 2
Principles and strategies of teaching part 2chinnex23
 
Banghay aralin sa edukasyon sa pagpapakatao
Banghay aralin sa edukasyon sa pagpapakataoBanghay aralin sa edukasyon sa pagpapakatao
Banghay aralin sa edukasyon sa pagpapakataoellaboi
 
Let prof ed reviewer
Let prof ed reviewerLet prof ed reviewer
Let prof ed reviewerbmsg wap
 
Assessment of Learning 1
Assessment of Learning 1Assessment of Learning 1
Assessment of Learning 1jn05
 

Was ist angesagt? (20)

900 questions gen ed
900 questions gen ed900 questions gen ed
900 questions gen ed
 
Assessment and evaluation of learning part 3
Assessment and evaluation of learning part 3Assessment and evaluation of learning part 3
Assessment and evaluation of learning part 3
 
Portfolio showcase
Portfolio showcasePortfolio showcase
Portfolio showcase
 
FLCT Chapter 4 Module.docx
FLCT Chapter 4 Module.docxFLCT Chapter 4 Module.docx
FLCT Chapter 4 Module.docx
 
Hagdang hagdang palayan
Hagdang hagdang palayanHagdang hagdang palayan
Hagdang hagdang palayan
 
Mga Likas na Yaman ng mga Lalawigan sa Rehiyon
Mga Likas na Yaman ng mga Lalawigan sa RehiyonMga Likas na Yaman ng mga Lalawigan sa Rehiyon
Mga Likas na Yaman ng mga Lalawigan sa Rehiyon
 
FS 1 Episode 3
FS 1 Episode 3FS 1 Episode 3
FS 1 Episode 3
 
LP in Grade 4- Science
LP in Grade 4- ScienceLP in Grade 4- Science
LP in Grade 4- Science
 
Earth sciencetrivia
Earth sciencetriviaEarth sciencetrivia
Earth sciencetrivia
 
Kpup re examined final2
Kpup re examined final2Kpup re examined final2
Kpup re examined final2
 
Yamang Tubig sa Pilipinas
Yamang Tubig sa PilipinasYamang Tubig sa Pilipinas
Yamang Tubig sa Pilipinas
 
Child and adolescent development part 2
Child and adolescent development part 2Child and adolescent development part 2
Child and adolescent development part 2
 
What is teaching internship?
What is teaching internship?What is teaching internship?
What is teaching internship?
 
254457273-LET-Review-Assessment-of-Learning-Test-Items.pptx
254457273-LET-Review-Assessment-of-Learning-Test-Items.pptx254457273-LET-Review-Assessment-of-Learning-Test-Items.pptx
254457273-LET-Review-Assessment-of-Learning-Test-Items.pptx
 
Principles and strategies of teaching part 3
Principles and strategies of teaching part 3Principles and strategies of teaching part 3
Principles and strategies of teaching part 3
 
Licensure examination for teachers1
Licensure examination for teachers1Licensure examination for teachers1
Licensure examination for teachers1
 
Principles and strategies of teaching part 2
Principles and strategies of teaching part 2Principles and strategies of teaching part 2
Principles and strategies of teaching part 2
 
Banghay aralin sa edukasyon sa pagpapakatao
Banghay aralin sa edukasyon sa pagpapakataoBanghay aralin sa edukasyon sa pagpapakatao
Banghay aralin sa edukasyon sa pagpapakatao
 
Let prof ed reviewer
Let prof ed reviewerLet prof ed reviewer
Let prof ed reviewer
 
Assessment of Learning 1
Assessment of Learning 1Assessment of Learning 1
Assessment of Learning 1
 

Andere mochten auch

General education set a with highlighted answers)
General education set a with highlighted answers)General education set a with highlighted answers)
General education set a with highlighted answers)Lucille Clavero
 
Math - analytic geometry
Math - analytic geometryMath - analytic geometry
Math - analytic geometryimmortalmikhel
 
Professional education set a (with highlighted answers)
Professional education set a (with highlighted answers)Professional education set a (with highlighted answers)
Professional education set a (with highlighted answers)Lucille Clavero
 
Professional education set b (with highlighted answers)
Professional education set b (with highlighted answers)Professional education set b (with highlighted answers)
Professional education set b (with highlighted answers)Lucille Clavero
 
General education
General educationGeneral education
General educationAlex Acayen
 
Professional education set d (with highlighted answers)
Professional education set d (with highlighted answers)Professional education set d (with highlighted answers)
Professional education set d (with highlighted answers)Lucille Clavero
 
LET Practice test in Professional Education-principles and strategies in teac...
LET Practice test in Professional Education-principles and strategies in teac...LET Practice test in Professional Education-principles and strategies in teac...
LET Practice test in Professional Education-principles and strategies in teac...University of Santo Tomas
 
General education
General educationGeneral education
General educationAlex_Track
 
Professional education set e (without highlighted answers)
Professional education set e (without highlighted answers)Professional education set e (without highlighted answers)
Professional education set e (without highlighted answers)Lucille Clavero
 
Let general education 3
Let general education 3Let general education 3
Let general education 3Alex Acayen
 
Let general education 9
Let general education 9Let general education 9
Let general education 9Alex Acayen
 
Let general education 2
Let general education 2Let general education 2
Let general education 2Alex Acayen
 
Professional education set d (without highlighted answers)
Professional education set d (without highlighted answers)Professional education set d (without highlighted answers)
Professional education set d (without highlighted answers)Lucille Clavero
 
The wedding dance lesson plan
The wedding dance lesson planThe wedding dance lesson plan
The wedding dance lesson planLucille Clavero
 
Jose ang batang magalang
Jose ang batang magalangJose ang batang magalang
Jose ang batang magalangBeth Reynoso
 
Intro to physical science and measurements
Intro to physical science and measurementsIntro to physical science and measurements
Intro to physical science and measurementssihellyay
 

Andere mochten auch (20)

General education set a with highlighted answers)
General education set a with highlighted answers)General education set a with highlighted answers)
General education set a with highlighted answers)
 
Math - analytic geometry
Math - analytic geometryMath - analytic geometry
Math - analytic geometry
 
General education set a
General education set aGeneral education set a
General education set a
 
Professional education set a (with highlighted answers)
Professional education set a (with highlighted answers)Professional education set a (with highlighted answers)
Professional education set a (with highlighted answers)
 
Professional education set b (with highlighted answers)
Professional education set b (with highlighted answers)Professional education set b (with highlighted answers)
Professional education set b (with highlighted answers)
 
General education
General educationGeneral education
General education
 
Professional education set d (with highlighted answers)
Professional education set d (with highlighted answers)Professional education set d (with highlighted answers)
Professional education set d (with highlighted answers)
 
LET Practice test in Professional Education-principles and strategies in teac...
LET Practice test in Professional Education-principles and strategies in teac...LET Practice test in Professional Education-principles and strategies in teac...
LET Practice test in Professional Education-principles and strategies in teac...
 
General education set b
General education set bGeneral education set b
General education set b
 
General education
General educationGeneral education
General education
 
Professional education set e (without highlighted answers)
Professional education set e (without highlighted answers)Professional education set e (without highlighted answers)
Professional education set e (without highlighted answers)
 
Let general education 3
Let general education 3Let general education 3
Let general education 3
 
Let general education 9
Let general education 9Let general education 9
Let general education 9
 
Let general education 2
Let general education 2Let general education 2
Let general education 2
 
Professional education set d (without highlighted answers)
Professional education set d (without highlighted answers)Professional education set d (without highlighted answers)
Professional education set d (without highlighted answers)
 
Series & Progression Mathematics
Series & Progression MathematicsSeries & Progression Mathematics
Series & Progression Mathematics
 
Ang bulag at ang pilay
Ang bulag at ang pilayAng bulag at ang pilay
Ang bulag at ang pilay
 
The wedding dance lesson plan
The wedding dance lesson planThe wedding dance lesson plan
The wedding dance lesson plan
 
Jose ang batang magalang
Jose ang batang magalangJose ang batang magalang
Jose ang batang magalang
 
Intro to physical science and measurements
Intro to physical science and measurementsIntro to physical science and measurements
Intro to physical science and measurements
 

Ähnlich wie General education set b with highlighted answers)

LET Sample Exam General education set b
LET Sample Exam General education set bLET Sample Exam General education set b
LET Sample Exam General education set bbmsg wap
 
Let general education 5
Let general education 5Let general education 5
Let general education 5Alex Acayen
 
LET Sample Exam General education set a
LET Sample Exam General education set aLET Sample Exam General education set a
LET Sample Exam General education set abmsg wap
 
3 tq gen ed
3 tq gen ed3 tq gen ed
3 tq gen edArneyo
 
Gen ed-answer-key-a
Gen ed-answer-key-aGen ed-answer-key-a
Gen ed-answer-key-aArneyo
 
LET reviewer in general education
LET reviewer in general educationLET reviewer in general education
LET reviewer in general educationmicrage
 
General Education LET or BLEPT reviewer
General Education LET or BLEPT reviewerGeneral Education LET or BLEPT reviewer
General Education LET or BLEPT reviewerelio dominglos
 
De thi-thu-thpt-quoc-gia-2015-mon-tieng-anh-truong-thpt-chuyen-bac-lieu
De thi-thu-thpt-quoc-gia-2015-mon-tieng-anh-truong-thpt-chuyen-bac-lieuDe thi-thu-thpt-quoc-gia-2015-mon-tieng-anh-truong-thpt-chuyen-bac-lieu
De thi-thu-thpt-quoc-gia-2015-mon-tieng-anh-truong-thpt-chuyen-bac-lieuonthitot .com
 
De thi-thu-thpt-quoc-gia-2015-mon-tieng-anh-truong-thpt-chuyen-bac-lieu
De thi-thu-thpt-quoc-gia-2015-mon-tieng-anh-truong-thpt-chuyen-bac-lieuDe thi-thu-thpt-quoc-gia-2015-mon-tieng-anh-truong-thpt-chuyen-bac-lieu
De thi-thu-thpt-quoc-gia-2015-mon-tieng-anh-truong-thpt-chuyen-bac-lieuHồng Nguyễn
 
CAT 1994 Previous Year Question Paper with Answer Key
CAT 1994 Previous Year Question Paper with Answer KeyCAT 1994 Previous Year Question Paper with Answer Key
CAT 1994 Previous Year Question Paper with Answer KeyEneutron
 
GEN ED 3.doc
GEN ED 3.docGEN ED 3.doc
GEN ED 3.docNoraima2
 
A better world
A better worldA better world
A better worlddlazcano
 
General Education - drill 6 - Part 1.docx
General Education - drill 6 - Part 1.docxGeneral Education - drill 6 - Part 1.docx
General Education - drill 6 - Part 1.docxShengHornalesOcampo
 
let reviewer and for college students.docx
let reviewer and for college students.docxlet reviewer and for college students.docx
let reviewer and for college students.docxxeinyenmoon
 
Let questions (3)
Let questions (3)Let questions (3)
Let questions (3)Arneyo
 
Let general education
Let general educationLet general education
Let general educationArneyo
 
General education
General educationGeneral education
General educationArneyo
 

Ähnlich wie General education set b with highlighted answers) (20)

Gen edu set b
Gen edu   set bGen edu   set b
Gen edu set b
 
LET Sample Exam General education set b
LET Sample Exam General education set bLET Sample Exam General education set b
LET Sample Exam General education set b
 
Let general education 5
Let general education 5Let general education 5
Let general education 5
 
LET Sample Exam General education set a
LET Sample Exam General education set aLET Sample Exam General education set a
LET Sample Exam General education set a
 
3 tq gen ed
3 tq gen ed3 tq gen ed
3 tq gen ed
 
Animal Farm Quiz #1
Animal Farm Quiz #1Animal Farm Quiz #1
Animal Farm Quiz #1
 
Gen ed-answer-key-a
Gen ed-answer-key-aGen ed-answer-key-a
Gen ed-answer-key-a
 
LET reviewer in general education
LET reviewer in general educationLET reviewer in general education
LET reviewer in general education
 
General Education Drills
General Education DrillsGeneral Education Drills
General Education Drills
 
General Education LET or BLEPT reviewer
General Education LET or BLEPT reviewerGeneral Education LET or BLEPT reviewer
General Education LET or BLEPT reviewer
 
De thi-thu-thpt-quoc-gia-2015-mon-tieng-anh-truong-thpt-chuyen-bac-lieu
De thi-thu-thpt-quoc-gia-2015-mon-tieng-anh-truong-thpt-chuyen-bac-lieuDe thi-thu-thpt-quoc-gia-2015-mon-tieng-anh-truong-thpt-chuyen-bac-lieu
De thi-thu-thpt-quoc-gia-2015-mon-tieng-anh-truong-thpt-chuyen-bac-lieu
 
De thi-thu-thpt-quoc-gia-2015-mon-tieng-anh-truong-thpt-chuyen-bac-lieu
De thi-thu-thpt-quoc-gia-2015-mon-tieng-anh-truong-thpt-chuyen-bac-lieuDe thi-thu-thpt-quoc-gia-2015-mon-tieng-anh-truong-thpt-chuyen-bac-lieu
De thi-thu-thpt-quoc-gia-2015-mon-tieng-anh-truong-thpt-chuyen-bac-lieu
 
CAT 1994 Previous Year Question Paper with Answer Key
CAT 1994 Previous Year Question Paper with Answer KeyCAT 1994 Previous Year Question Paper with Answer Key
CAT 1994 Previous Year Question Paper with Answer Key
 
GEN ED 3.doc
GEN ED 3.docGEN ED 3.doc
GEN ED 3.doc
 
A better world
A better worldA better world
A better world
 
General Education - drill 6 - Part 1.docx
General Education - drill 6 - Part 1.docxGeneral Education - drill 6 - Part 1.docx
General Education - drill 6 - Part 1.docx
 
let reviewer and for college students.docx
let reviewer and for college students.docxlet reviewer and for college students.docx
let reviewer and for college students.docx
 
Let questions (3)
Let questions (3)Let questions (3)
Let questions (3)
 
Let general education
Let general educationLet general education
Let general education
 
General education
General educationGeneral education
General education
 

Kürzlich hochgeladen

Science 7 Quarter 4 Module 2: Natural Resources.pptx
Science 7 Quarter 4 Module 2: Natural Resources.pptxScience 7 Quarter 4 Module 2: Natural Resources.pptx
Science 7 Quarter 4 Module 2: Natural Resources.pptxMaryGraceBautista27
 
Inclusivity Essentials_ Creating Accessible Websites for Nonprofits .pdf
Inclusivity Essentials_ Creating Accessible Websites for Nonprofits .pdfInclusivity Essentials_ Creating Accessible Websites for Nonprofits .pdf
Inclusivity Essentials_ Creating Accessible Websites for Nonprofits .pdfTechSoup
 
Difference Between Search & Browse Methods in Odoo 17
Difference Between Search & Browse Methods in Odoo 17Difference Between Search & Browse Methods in Odoo 17
Difference Between Search & Browse Methods in Odoo 17Celine George
 
ECONOMIC CONTEXT - PAPER 1 Q3: NEWSPAPERS.pptx
ECONOMIC CONTEXT - PAPER 1 Q3: NEWSPAPERS.pptxECONOMIC CONTEXT - PAPER 1 Q3: NEWSPAPERS.pptx
ECONOMIC CONTEXT - PAPER 1 Q3: NEWSPAPERS.pptxiammrhaywood
 
Karra SKD Conference Presentation Revised.pptx
Karra SKD Conference Presentation Revised.pptxKarra SKD Conference Presentation Revised.pptx
Karra SKD Conference Presentation Revised.pptxAshokKarra1
 
How to Add Barcode on PDF Report in Odoo 17
How to Add Barcode on PDF Report in Odoo 17How to Add Barcode on PDF Report in Odoo 17
How to Add Barcode on PDF Report in Odoo 17Celine George
 
call girls in Kamla Market (DELHI) 🔝 >༒9953330565🔝 genuine Escort Service 🔝✔️✔️
call girls in Kamla Market (DELHI) 🔝 >༒9953330565🔝 genuine Escort Service 🔝✔️✔️call girls in Kamla Market (DELHI) 🔝 >༒9953330565🔝 genuine Escort Service 🔝✔️✔️
call girls in Kamla Market (DELHI) 🔝 >༒9953330565🔝 genuine Escort Service 🔝✔️✔️9953056974 Low Rate Call Girls In Saket, Delhi NCR
 
ACC 2024 Chronicles. Cardiology. Exam.pdf
ACC 2024 Chronicles. Cardiology. Exam.pdfACC 2024 Chronicles. Cardiology. Exam.pdf
ACC 2024 Chronicles. Cardiology. Exam.pdfSpandanaRallapalli
 
Q4 English4 Week3 PPT Melcnmg-based.pptx
Q4 English4 Week3 PPT Melcnmg-based.pptxQ4 English4 Week3 PPT Melcnmg-based.pptx
Q4 English4 Week3 PPT Melcnmg-based.pptxnelietumpap1
 
Roles & Responsibilities in Pharmacovigilance
Roles & Responsibilities in PharmacovigilanceRoles & Responsibilities in Pharmacovigilance
Roles & Responsibilities in PharmacovigilanceSamikshaHamane
 
ENGLISH6-Q4-W3.pptxqurter our high choom
ENGLISH6-Q4-W3.pptxqurter our high choomENGLISH6-Q4-W3.pptxqurter our high choom
ENGLISH6-Q4-W3.pptxqurter our high choomnelietumpap1
 
Gas measurement O2,Co2,& ph) 04/2024.pptx
Gas measurement O2,Co2,& ph) 04/2024.pptxGas measurement O2,Co2,& ph) 04/2024.pptx
Gas measurement O2,Co2,& ph) 04/2024.pptxDr.Ibrahim Hassaan
 
MULTIDISCIPLINRY NATURE OF THE ENVIRONMENTAL STUDIES.pptx
MULTIDISCIPLINRY NATURE OF THE ENVIRONMENTAL STUDIES.pptxMULTIDISCIPLINRY NATURE OF THE ENVIRONMENTAL STUDIES.pptx
MULTIDISCIPLINRY NATURE OF THE ENVIRONMENTAL STUDIES.pptxAnupkumar Sharma
 
USPS® Forced Meter Migration - How to Know if Your Postage Meter Will Soon be...
USPS® Forced Meter Migration - How to Know if Your Postage Meter Will Soon be...USPS® Forced Meter Migration - How to Know if Your Postage Meter Will Soon be...
USPS® Forced Meter Migration - How to Know if Your Postage Meter Will Soon be...Postal Advocate Inc.
 
Computed Fields and api Depends in the Odoo 17
Computed Fields and api Depends in the Odoo 17Computed Fields and api Depends in the Odoo 17
Computed Fields and api Depends in the Odoo 17Celine George
 
ANG SEKTOR NG agrikultura.pptx QUARTER 4
ANG SEKTOR NG agrikultura.pptx QUARTER 4ANG SEKTOR NG agrikultura.pptx QUARTER 4
ANG SEKTOR NG agrikultura.pptx QUARTER 4MiaBumagat1
 

Kürzlich hochgeladen (20)

Science 7 Quarter 4 Module 2: Natural Resources.pptx
Science 7 Quarter 4 Module 2: Natural Resources.pptxScience 7 Quarter 4 Module 2: Natural Resources.pptx
Science 7 Quarter 4 Module 2: Natural Resources.pptx
 
Inclusivity Essentials_ Creating Accessible Websites for Nonprofits .pdf
Inclusivity Essentials_ Creating Accessible Websites for Nonprofits .pdfInclusivity Essentials_ Creating Accessible Websites for Nonprofits .pdf
Inclusivity Essentials_ Creating Accessible Websites for Nonprofits .pdf
 
LEFT_ON_C'N_ PRELIMS_EL_DORADO_2024.pptx
LEFT_ON_C'N_ PRELIMS_EL_DORADO_2024.pptxLEFT_ON_C'N_ PRELIMS_EL_DORADO_2024.pptx
LEFT_ON_C'N_ PRELIMS_EL_DORADO_2024.pptx
 
Difference Between Search & Browse Methods in Odoo 17
Difference Between Search & Browse Methods in Odoo 17Difference Between Search & Browse Methods in Odoo 17
Difference Between Search & Browse Methods in Odoo 17
 
ECONOMIC CONTEXT - PAPER 1 Q3: NEWSPAPERS.pptx
ECONOMIC CONTEXT - PAPER 1 Q3: NEWSPAPERS.pptxECONOMIC CONTEXT - PAPER 1 Q3: NEWSPAPERS.pptx
ECONOMIC CONTEXT - PAPER 1 Q3: NEWSPAPERS.pptx
 
Karra SKD Conference Presentation Revised.pptx
Karra SKD Conference Presentation Revised.pptxKarra SKD Conference Presentation Revised.pptx
Karra SKD Conference Presentation Revised.pptx
 
How to Add Barcode on PDF Report in Odoo 17
How to Add Barcode on PDF Report in Odoo 17How to Add Barcode on PDF Report in Odoo 17
How to Add Barcode on PDF Report in Odoo 17
 
call girls in Kamla Market (DELHI) 🔝 >༒9953330565🔝 genuine Escort Service 🔝✔️✔️
call girls in Kamla Market (DELHI) 🔝 >༒9953330565🔝 genuine Escort Service 🔝✔️✔️call girls in Kamla Market (DELHI) 🔝 >༒9953330565🔝 genuine Escort Service 🔝✔️✔️
call girls in Kamla Market (DELHI) 🔝 >༒9953330565🔝 genuine Escort Service 🔝✔️✔️
 
ACC 2024 Chronicles. Cardiology. Exam.pdf
ACC 2024 Chronicles. Cardiology. Exam.pdfACC 2024 Chronicles. Cardiology. Exam.pdf
ACC 2024 Chronicles. Cardiology. Exam.pdf
 
Raw materials used in Herbal Cosmetics.pptx
Raw materials used in Herbal Cosmetics.pptxRaw materials used in Herbal Cosmetics.pptx
Raw materials used in Herbal Cosmetics.pptx
 
Q4 English4 Week3 PPT Melcnmg-based.pptx
Q4 English4 Week3 PPT Melcnmg-based.pptxQ4 English4 Week3 PPT Melcnmg-based.pptx
Q4 English4 Week3 PPT Melcnmg-based.pptx
 
Roles & Responsibilities in Pharmacovigilance
Roles & Responsibilities in PharmacovigilanceRoles & Responsibilities in Pharmacovigilance
Roles & Responsibilities in Pharmacovigilance
 
ENGLISH6-Q4-W3.pptxqurter our high choom
ENGLISH6-Q4-W3.pptxqurter our high choomENGLISH6-Q4-W3.pptxqurter our high choom
ENGLISH6-Q4-W3.pptxqurter our high choom
 
Gas measurement O2,Co2,& ph) 04/2024.pptx
Gas measurement O2,Co2,& ph) 04/2024.pptxGas measurement O2,Co2,& ph) 04/2024.pptx
Gas measurement O2,Co2,& ph) 04/2024.pptx
 
MULTIDISCIPLINRY NATURE OF THE ENVIRONMENTAL STUDIES.pptx
MULTIDISCIPLINRY NATURE OF THE ENVIRONMENTAL STUDIES.pptxMULTIDISCIPLINRY NATURE OF THE ENVIRONMENTAL STUDIES.pptx
MULTIDISCIPLINRY NATURE OF THE ENVIRONMENTAL STUDIES.pptx
 
USPS® Forced Meter Migration - How to Know if Your Postage Meter Will Soon be...
USPS® Forced Meter Migration - How to Know if Your Postage Meter Will Soon be...USPS® Forced Meter Migration - How to Know if Your Postage Meter Will Soon be...
USPS® Forced Meter Migration - How to Know if Your Postage Meter Will Soon be...
 
Computed Fields and api Depends in the Odoo 17
Computed Fields and api Depends in the Odoo 17Computed Fields and api Depends in the Odoo 17
Computed Fields and api Depends in the Odoo 17
 
OS-operating systems- ch04 (Threads) ...
OS-operating systems- ch04 (Threads) ...OS-operating systems- ch04 (Threads) ...
OS-operating systems- ch04 (Threads) ...
 
YOUVE GOT EMAIL_FINALS_EL_DORADO_2024.pptx
YOUVE GOT EMAIL_FINALS_EL_DORADO_2024.pptxYOUVE GOT EMAIL_FINALS_EL_DORADO_2024.pptx
YOUVE GOT EMAIL_FINALS_EL_DORADO_2024.pptx
 
ANG SEKTOR NG agrikultura.pptx QUARTER 4
ANG SEKTOR NG agrikultura.pptx QUARTER 4ANG SEKTOR NG agrikultura.pptx QUARTER 4
ANG SEKTOR NG agrikultura.pptx QUARTER 4
 

General education set b with highlighted answers)

  • 1. Page 1 of 55 GEN ED SET B 1. Alin ang kahulugan ng AGAW-BUHAY? A. Masiglang-masigla B. Malapit nang mamatay C. Pagpapatuloy ng buhay D. Mahirap na buhay 2. Robert Frost wrote the poemAcquainted with the Nightfrom which the stanza is taken: I have been one acquainted with the night. I have walked out in rain-back in rain. I have out walked the farthest city light The poet in the stanza talks of A. isolation and loneliness. B. happiness in having been acquainted with the night. C. joy getting out of the house. D. youthful delight playing in the rain. 3. The following are defects present at the time of marriage which is voidable and annullable EXCEPT A. impotence B. deceit C. fraud D. threat 4. What does a stick of unlit dynamite demonstrate? A. Chemical Energy B. Nuclear Energy C. Kinetic Energy D. Potential Energy
  • 2. Page 2 of 55 5. Ang sumusunod ay isang talumpati na may pamagat na SA KABATAAN na sinulat ni Onofre Pagsanghan. Isa sa mga salitang napag-aralan natin sa wikang Pilipino ay ang salitang NABANSOT. Kapang ang isang bagay daw ay dapat pang lumaki ngunit ito'y tumigil na sa paglaki, ang bagay na ito raw ay NABANSOT. Marami raw uri ng pagkabansot ngunit ay pinakamalungkot na uri raw ay ang pagkabansot ng isipan, ng puso at ng diwa. Ang panahon ng kabataan ay panahon ng paglaki, ngunit ang ating paglaki ay kailangang paglaki, at pag-unlad ng ating buong katauhan, hindi lamang ng ating sukat at timbang. Kung ga-poste man ang ating taas at ga-pison man ang ating bigat ngunit kung ang pag-iisip naman nati'y ga-kulisap lamang kay pangit na kabansutan. Kung tumangkad man tayong tangkad-kawayan at bumilog man tayong bilog-tapayan, ngunit kung tayo nama'y tulad nibondyingay di mapagkatiwalaan-anong laking kakulangan. Kung magkakatawan tayong katawangTarzanat mapatalas ang ating isipang sintalas ng kay Rizal, ngunit kung ang ating kalooban nama'y itim na duwende ng kasamaan-anong kapinsalaan para sa kinabukasan. Kinabukasan, kabataan, tayo raw ang pag-asa ng Inang Bayan. Tayo raw ang maghahatid sa kanya sa langit ng kasaganaan at karangalan o hihils sa kanya sa putik ng kahirapan at kahihiyan. Ang panahon ng pagkilos ay ngayon, hindi bukas, hindi sa isang taon. Araw-araw ay tumutuwid tayong palangit or bumabaluktod tayong paputik. Tamang-tama ang sabi ng ating mga ninunong kung ano raw ang kinamihasnan ay siyang pagkakatandaan. Huwag nating akalaing makapagpapabaya tayo ng ating pag-aaral ngayon at sa araw ng bukas ay bigla tayong magiging mga dalubhasang magpapaunlad sa bayan. Huwag nating akalaing makapaglulublob tayo ngayon sa kalaswaaan at kahalayan at sa mahiwagang araw ng bukas sigla tayong magiging ulirang mga magulang. Kabataan, tunay na pag-ibig sa bayan, ang tunay na nasyonalismo, ay wala sa tamis ng pangarap wala rin sa pagpag ng dila. Ang tunay na pag-ibig ay nasa pawis ng gawa. Alin sa mga sumusunod ang mensahe ng talumpati? A. Ang mataas na paniniwala at taimtim na pananalig ay kailangang taglayin upang ang hangarin sa buhay ay ating kamtin. B. Ang panahon ng kabataan ay panahon ng paglaki at pagbabagong makabuluhan. C. Ang gawa ang siyang sukat ng kadakilaan. D. Ang kabataan ay siyang pag-asa ng bayan. 6. Ano ang kahulugan ng taludtod na ito? Ang anak man ay alagaan sa marubdob na pagsuyo sikapin ma sa sarili'y huwag siyang maging luko talipandas sa paglaki na sa sama marahuyo sa lahi mo't sa Bayan may isang tinik sa balaho. A. Mahalin ang anak ng walang hangganan. B. Tamang pagpapalaki sa anak ang dapat.
  • 3. Page 3 of 55 C. Suyuin ang anak at ibigay lahat ng hilig. D. Paligayahin ang tahanan. 7. The carat is a unit of measure used to weigh precious stones. It equals 3.086 grains. How many grains does a 2.8 carat diamond weigh? A. 864.08 B. 86.408 C. 8.6408 D. 8640.8 E. none of these 8. Alin ang kahulugan ng KAHIRAMANG SUKLAY? A. Kakilala B. Kaibigan C. Karibal D. Kalahi 9. Nasa anong kaganapan ng pandiwa ang pangungusap? Naglaro ng basketball sa Rizal Stadium ang koponan ng aming pamantasan. A. Sanhi B. Tagaganap C. Kagamitan D. Ganapan 10. Julieta Villaruel was a landowner from Cabiao, Nueva Ecija. Under the CARL, she was claiming 8 hectares, 5 hectares of which represented the retention limit and the 3 hectares for her only child. Why was her child denied the 3 hectares? A. Her son was 15 years old who was actually tiling the farm B. Her son was 17 years old who was managing the farm C. Her son was 13 years old who has been helping till the farm D. Her son was 19 years old who was actually tilling or managing the farm
  • 4. Page 4 of 55 11. Sa Espiritu ni Bathala ang nangangalaga ng kanilang kalusugan ang ipinahihiwatig na katangian ay __________. A. malinis B. mabisa C. maliksi D. makapangyarihan 12.Find m in the proportion m/12 = 30/24. A. 30 B. 15 C. 20 D. 25 E. none of these 13. One of the most outstanding accomplishments of the cooperative movement is the encouragement of thrift. Which maxim of God puts this into practice? A. God helps those who help themselves B. Look at the birds: they do not plant seeds,/gather a harvest and put it in barns; yet your Father in heaven takes care of them! C. He is near to those who call to Him, who call to Him with sincerity D. Happy are those who are merciful to others; God will be merciful to them! 14. Which one is the human right to life? A. Peace B. Live in national and international order C. Own property D. Fair trial 15. Which antidote would have a similar effect if vinegar or citrus juice were not available? A. Milk B. Raw egg white
  • 5. Page 5 of 55 C. Vegetable oil D. Water 16. Thousands of street children in large Brazilian cities were murdered by paramilitary death squads which included police officers. What could be the reason why these operations were not suppressed by the government? A. Totalitarian governments do not give protection to their people. B. The business people even funded these operations to clean up their streets and neighborhoods. C. The Universal Declaration of Human Rights was only lip service. D. To how they treat their people was nobody else's business. 17. What is the difference between the largest 4-digit number and the smallest 4-digit number? A. 8999 B. 8000 C. 9998 D. 8888 E. none of these 18. Si Mariano Ponce ay propagandistang may sagisag sa panulat na A. Tamaraw B. Tikbalang C. Kapre D. Kalapate 19. Below is a biographical sketch. Success often comes to those with humble beginnings, Elvis Aron Presley was born on January 8, 1935, in Tupelo, Mississippi. He first sang in a church choir and taught himself to play the guitar, but he never learned to read music. By 1953, he had moved to Memphis, Tennessee, graduated from high school, and enrolled in night school to become an electrician. That year, at Sun Records, Presley recorded a personal record for his mother, a song that was heard by the company's president. As a result of the president's recognition, Presley's first record,That's all Right, Mama,was out in 1954. He toured the South, and in 1955 five of his records were released simultaneously. His first national television appearance was that year on Jackie Gleason'sThe Stage Show,but Presley became known for his appearance onThe Ed Sullivan Show,where the young singer gyrated as he sangrock n'
  • 6. Page 6 of 55 rollmusic. During the live television performance, Presley was photographed only from the waist up because his motions were considered obscene. Elvis the Pelvisbegan his film career in 1956 with LOVE ME TENDER and signed a long term film contract. The movie critics were not always kind, but teenagers flocked to Presley's films. Within a few short years, Presley had established a career that would span twenty-five years of ups and downs and make him one of the most popular entertainers in history. Long after his untimely death at age 42, Presley would be remembered asThe King of Rock n' Roll The main idea of the sketch is that A. singers are more successful if they appear in films B. there has always been obscenity on television C. opportunity and luck are often as important as hard work D. celebrities are usually more famous after their death 20. What do the following lines CONVEY? Midnight, not a sound from the pavement. Has the moon lost her memory? She is smiling alone. In the lamp light the withered leaves Collect at my feet And the wind begins to moan A. Confusion B. Optimism C. Loneliness D. Eagerness 21. Anong uri ng panghalip ang salitang may salungguhit sa pangungusap? May padalang tulong ang pamahalaan para sa kanila. A. Pambalana B. Palagyo C. Paari D. Palayon 22. Alin ang di-karaniwang anyo ng pandiwang WINIKA KO? A. Ikako
  • 7. Page 7 of 55 B. Wikako C. Kako D. Wika ko 23. Why does a pendulum in a grandfather clock once set in motion continue to swing, thereby regulating the clock's movement? This is due to the Law of A. Universal Gravitation B. Action and Reaction C. Applied Force D. Inertia 24. Which of the following BEST demonstrates the greenhouse principle? A. A heated aquarium B. A car with rolled-up windows C. A microwave oven D. A solar battery -powered calculator 25. Which one explains why oxygen a gas, is the largest component of the Earth’s crust? A. Oxygen gives Earth's crust its lightness B. Oxygen is the most abundant element in the world C. Oxygen is capable of combining with most of the elements in the Earth's crust. D. Oxygen is needed to sustain all life on Earth. 26. Which is the BEST WAY to write the underlined portion of this sentence? Many viewers taped shows to watch later. A. tapped B. had taped C. tape D. had tapped
  • 8. Page 8 of 55 27. Below is an excerpt from John F. Kennedy's "Inaugural Address." In your hands, my fellow citizens, more than mine, will rest the final success or failure of our course. Since this country as founded, each generation of Americans has been summoned to give testimony to its national loyalty. The graves of young Americans into answered the call to service surround the globe. Now the trumpet summons us again-not as a call to bear arms, though, arms we need, not as a call to battle, though embattled we are; but a call to bear the burden of a long twilight struggle, year in and year out, rejoicing in hope, patient in tribulation, a, struggle against the common enemies of man: tyranny, poverty, disease, and war itself. Can we forge against these enemies a grand and global alliance, North and South, East and West, that can assure a more fruitful life for all mankind? Will you join me in this historic effort? In the long history of the world, only a few generations have been granted the role of defending freedom in its hour of maximum danger. I do not shrink from this responsibility; I welcome it I do not believe that any of us would exchange places with any other people or any other generation. The energy, the faith, the devotion which we bring to this endeavor will light our country and all who serve it, and the glow from that fire can truly light the world. And so, my fellow Americans, ask not what your country can do for you; ask what you can do for your country. My Fellow citizens of the world ask not what America will do for you, but what together we can do for the freedom of man. The tone of the speech can BEST be characterized as A. the use of the personal pronouns we and us to build rapport with listeners B. catchy turns of phrase in which subjects and objects are inverted C. a standard, predictable rhythm and the use of rhyme D. the repetition of key words 28. Sa alin salita magkakaroon ng saglit na paghinto kung pinapilitang si Rose ang nakabasag ng pinggan? Hindi si Rose ang nakabasag ng pinggan. A. Rose B. Hindi C. Nakabasag D. Pinggan 29. Below is the poem written by Edgar Lee Masters in 1915: What Happened to Tom Merritt? At first I suspected something She acted so calm and absent-minded. And one day I heard the back door shut,
  • 9. Page 9 of 55 As I entered the front, and I saw him slink Back of the smoke house into the lot, And across the field. And I meant to kill him on sight. But that day, walking near Fourth Bridge, Without a stick or a stone at hand, All of a sudden I saw him standing, Scared to death, holding his rabbits, And all l could say was,Don't, Don't, Don't, As he aimed and fired at my heart. The way in which the poet presents these words in line 12 implies that Tom A. tried to annoy the other man. B. was shot before the finished the statement. C. did not want to hurt the other man. D. begged the man to stop seeing his wife. 30. Carter's part in relinquishing U.S. control of the Canal Zone to Panama is described as a victory for __________. A. conservatism B. anti-imperialism C. isolationism D. imperialism 31. How many whole numbers can divide 30 exactly? A. Eight B. Six C. Five D. Four E. none of these 32. Which are the next three terms in the progression 1, 4, 16 . . . 8 terms? A. 64, 256, 1024 B. 67, 259, 1027 C. 66, 258, 1026
  • 10. Page 10 of 55 D. 65, 257, 1025 E. none of these 33. Which is NOT personal integrity? A. Time B. Place C. Order D. Harmony 34. One package is 100 pounds, and the other is 150 pounds. The weight of the second package is how many times that of the first? A. 1½ times heavier B. ½as heavy C. none of these D. 10 pounds heavier E. 20 pounds heavier 35. If a baseball player hits 10 home runs in the first 45 games, at the same rate how many home runs can he expect to hit during the 162-game season? A. 38 B. 42 C. 36 D. 40 E. none of these 36. Of the following changes in the socio-economic, political cultural and physical that have occurred in the Filipino family, which one remains to be TRUE? A. The loss of the traditional evening prayer and the ritual of blessing (mano) B. The unity of the family despite competing demands
  • 11. Page 11 of 55 C. The continued parental influence over children's language dress and other behavior D. The continued support for parents and siblings 37. What day follows the day before yesterday if 2 days from now will be Sunday? A. Tuesday B. none of these C. Saturday D. Wednesday E. Thursday 38. What correction should be made to this sentence? One of their theories is that the first child receives more of the parents' attention than other children so first-borns tend to be more intellectual. A. Change is to are B. Insert a comma after children C. Change parents' to parent's D. Change theories to theory's 39. The poem below is entitledSuburban Prophecywhich is written by Howard Nemerov. On Saturday, the power-mowers' whine Begins the morning. Over this neighbourhood Rises the keening, petulant voice, begin Green oily teeth to chatter and munch the cud. Monsters, crawling the carpets of the world, Still send from underground against your blades The roots of things battalions green and curled And tender, that will match your blades with blades Till the revolted throats shall strangle on. The tickle of their dead, till straws shall break Crankshafts like camels, and the sun go down On dinosaurs in swamps. A night attack Follows and by the time the Sabbath dawns All armored beasts are eaten by their lawns. How long does the action of the poem take place?
  • 12. Page 12 of 55 A. A week B. Twenty-four horns C. An afternoon D. A morning 40. A meter was cut at the 35-cm mark. What is the ratio of the smaller piece to the larger piece? A. 7:13 B. 65:35 C. 35:100 D. 65:100 E. none of these 41. Below is an excerpt from John F. Kennedy's "Inaugural Address." In your hands, my fellow citizens, more than mine, will rest the final success or failure of our course. Since this country as founded, each generation of Americans has been summoned to give testimony to its national loyalty. The graves of young Americans into answered the call to service surround the globe. Now the trumpet summons us again-not as a call to bear arms, though, arms we need, not as a call to battle, though embattled we are; but a call to bear the burden of a long twilight struggle, year in and year out, rejoicing in hope, patient in tribulation, a, struggle against the common enemies of man: tyranny, poverty, disease, and war itself. Can we forge against these enemies a grand and global alliance, North and South, East and West, that can assure a more fruitful life for all mankind? Will you join me in this historic effort? In the long history of the world, only a few generations have been granted the role of defending freedom in its hour of maximum danger. I do not shrink from this responsibility; I welcome it I do not believe that any of us would exchange places with any other people or any other generation. The energy, the faith, the devotion which we bring to this endeavor will light our country and all who serve it, and the glow from that fire can truly light the world. And so, my fellow Americans, ask not what your country can do for you; ask what you can do for your country. My Fellow citizens of the world ask not what America will do for you, but what together we can do for the freedom of man. In the speech, Kennedy paints a picture of the United States as a nation that is A. longing to return to the past B. on the brink of world war C. struggling to survive D. the leading defender of freedom
  • 13. Page 13 of 55 42. Which of the following will occur if a cold bottle of soda is left open on a kitchen counter? A. The pressure that the soda exerts on the bottle will increase. B. The temperature of the soda will decrease. C. The amount of dissolved carbon dioxide gas will decrease. D. The amount of dissolved carbon dioxide gas will remain the same. 43. In how many ways can you arrange three mathematics books (Algebra, Geometry, Trigonometry) in order on a shelf? A. 6 B. 8 C. 12 D. 24 E. none of these 44. What are the next four numbers in this sequence 8, 5, 4, 9,17, ___, ___, ___, ___? A. 4, 3, 2, 1 B. 5, 4, 3, 2 C. 6, 3, 2, 0 D. 3, 2, 1, 0 E. none of these 45. A politician wants to get his message to 2/3 of the population of 48,000 in Bulacan. However, his advertising campaign reaches only 3/4 of the number he intended. How many people does he actually reach? A. 16,000 B. 10,000 C. 24,000 D. 36,000 E. none of these 46. Aling antas ng tona ang lumilitaw sa bahaging may salungguhit ng pangungusap na nagdududa?
  • 14. Page 14 of 55 Nagpuputol ng puno ang lalaki. A. 1 B. 2 C. 4 D. 3 47. Which is NOT among the hazardous effects of water pollution to health? A. The epidemic threat of hepatitis and dysentery B. The increase incidence of liver cancer C. The dumping of mercury in the sea causing blindness, brain damage, or death D. The presence of certain bacteria in the digestive tract causing methemoglobinemia 48. Aling antas ng tono ang lumilitaw sa bahaging may salungguhit ng pangungusap na nagsasalaysay? Magbabasa ng mga gawain ang guro. A. 1 B. 3 C. 2 D. 4 49. A tightly coiled spring demonstrates? A. Steam energy B. Kinetic energy C. Potential energy D. Chemical energy 50. Ano ang pinakaangkop na kahulugan nito? Nagsasaya tayo ngayon sapagkat ang iyong namatay na kapatid ay muling nabuhay; ang nawawala
  • 15. Page 15 of 55 ay muling nakita. A. Ang pagbabalik ay dapat ipaghanda nang malaki. B. Ang pagbabago ng kapatid ay dapat pahalagahan. C. Dapat silang magsaya sa muli nilang pagsasama-sama D. Ang pagsasama nila ay dahil sa muling pagbabalik ng kapatid. 51. The poem below is entitledSuburban Prophecywhich is written by Howard Nemerov. On Saturday, the power-mowers' whine Begins the morning. Over this neighbourhood Rises the keening, petulant voice, begin Green oily teeth to chatter and munch the cud. Monsters, crawling the carpets of the world, Still send from underground against your blades The roots of things battalions green and curled And tender, that will match your blades with blades Till the revolted throats shall strangle on. The tickle of their dead, till straws shall break Crankshafts like camels, and the sun go down On dinosaurs in swamps. A night attack Follows and by the time the Sabbath dawns All armored beasts are eaten by their lawns. The imagery in the first stanza appeals to the reader's sense of A. sight B. touch C. smell D. hearing 52. Which part of the Allied action has been detailed in this passage? After Saddam Hussein violated international agreements by sending Iraq troops to Kuwait and missiles into other neighboring countries, the Allies responded with military action. A. The Allied bases in Saudi Arabia B. The ground was in the desert C. The movement of Allied Troops with Iraq D. The air campaign in the Baghdad area
  • 16. Page 16 of 55 53. Which of the following words DOES NOT contain the voiceless Th? A. Mouth B. Breath C. Teeth D. Health 54. Which of the following is the BEST example of self-preservation? A. mouse runs when it sees a cat. B. A dog barks when it sees its owner. C. A young man decides to quit smoking D. A salmon swims back to the place of its birth to lay eggs. 55. Carl Sundburg wrote "Jazz Fantasia" which has for its first stanza: Drum on your drums, batter on your banjos, sob on the long cool winding saxophones. Go to it, O jazzmen. Which words illustrate alliteration? A. Batter and banjos B. Long and cool C. Sob and winding D. To and it 56. Which of the following lines is a simile? A. Holding wonder like a cup B. Life has loneliness to sell C. Eyes that love you, arms that hold D. Buy it and never count the cost 57. What is the MOST likely reason for a desert plant to have a few or no leaves? A. To increase photosynthesis B. To decrease photosynthesis
  • 17. Page 17 of 55 C. To increase transpiration D. To decrease transpiration 58. Which are limited only to the sale of real property and stock transaction? A. Business incomes B. Capital gains C. Employment incomes D. Passive incomes 59. What is the mood of these lines? Daylight, I must wait for the sunrise I must think of a new life And I mustn't give in. When the dawn comes tonight will be a memory, too And a new day will begin. A. Afraid B. Sarcastic C. Depressed D. Hopeful 60. Below is an excerpt from John F. Kennedy's "Inaugural Address." In your hands, my fellow citizens, more than mine, will rest the final success or failure of our course. Since this country as founded, each generation of Americans has been summoned to give testimony to its national loyalty. The graves of young Americans into answered the call to service surround the globe. Now the trumpet summons us again-not as a call to bear arms, though, arms we need, not as a call to battle, though embattled we are; but a call to bear the burden of a long twilight struggle, year in and year out, rejoicing in hope, patient in tribulation, a, struggle against the common enemies of man: tyranny, poverty, disease, and war itself. Can we forge against these enemies a grand and global alliance, North and South, East and West, that can assure a more fruitful life for all mankind? Will you join me in this historic effort? In the long history of the world, only a few generations have been granted the role of defending freedom in its hour of maximum danger. I do not shrink from this responsibility; I welcome it I do not believe that any of us would exchange places with any other people or any other generation. The energy, the faith, the devotion which we bring to this endeavor will light our country and all who serve it, and the glow from that fire can truly light the world. And so, my fellow Americans, ask not what your country can do for you; ask what you can do for your country. My Fellow citizens of the world ask not what America will do for you, but what together we can do for
  • 18. Page 18 of 55 the freedom of man. The speech is characterized by all of the following stylistic devices EXCEPT A. sad B. uplifting C. light-hearted D. sarcastic 61. How many ways can a committee of 4 people be selected from a group of 7 people? A. 35 B. 70 C. 140 D. 210 E. none of these 62. Which word contains the voiced Th? A. Thank B. These C. Think D. Thing 63. Which is the BEST WAY to write the underlined portion of this sentence? However, if a fire extinguisher is handy, a quick-thinking person often can use them to put out a small fire. A. they B. him C. them D. it 64. What indoor relative humidity range would probably be comfortable when the outside temperature and humidity levels are extremely low? A. 90 to 100% B. 50 to 70%
  • 19. Page 19 of 55 C. 20 to 30% D. 30 to 40% 65. What do you predict will happen when you bring two bar magnets closer together? A. They will repel each other. B. They will create an alternating current C. Nothing will happen D. They will attract each other. 66. Kaninong tula hango ang sumusunod? Ang hindi magmahal sa sariling wika mahigit sa hayop at malansang isda. A. Jose Rizal B. Emilio Jacinta C. Apolinario Mabini D. Graciano Lopez Jaena 67. Which word is read on a high note to describe the kind of day? This is a cold day. A. is B. cold C. day D. this 68. The Jones family has had four children, all girls. The fifth child born is a boy. This change is the result of A. conception classes taken by the parents. B. the timing of the fertility cycles. C. the father's contribution of a "Y" chromosome. D. the "law of averages" finally catching up. 69. Which one is the right to human dignity?
  • 20. Page 20 of 55 A. Choose the goals and means of development B. Share in scientific and technological advances of the world C. Right to information D. Sovereignty over our natural resources 70. Which is the BEST WAY to write the underlined portion of this sentence? Researchers also speculate that some teachers might have given boys more computer time because parents and teachers expected, boys to need computers for future careers. A. will expect B. expected C. will have expected D. expecting 71. Below is a biographical sketch. Success often comes to those with humble beginnings, Elvis Aron Presley was born on January 8, 1935, in Tupelo, Mississippi. He first sang in a church choir and taught himself to play the guitar, but he never learned to read music. By 1953, he had moved to Memphis, Tennessee, graduated from high school, and enrolled in night school to become an electrician. That year, at Sun Records, Presley recorded a personal record for his mother, a song that was heard by the company's president. As a result of the president's recognition, Presley's first record,That's all Right, Mama,was out in 1954. He toured the South, and in 1955 five of his records were released simultaneously. His first national television appearance was that year on Jackie Gleason'sThe Stage Show,but Presley became known for his appearance onThe Ed Sullivan Show,where the young singer gyrated as he sangrock n' rollmusic. During the live television performance, Presley was photographed only from the waist up because his motions were considered obscene. Elvis the Pelvisbegan his film career in 1956 with LOVE ME TENDER and signed a long term film contract. The movie critics were not always kind, but teenagers flocked to Presley's films. Within a few short years, Presley had established a career that would span twenty-five years of ups and downs and make him one of the most popular entertainers in history. Long after his untimely death at age 42, Presley would be remembered asThe King of Rock n' Roll The last sentence reveals that the author's attitude toward Presley is one of A. indifference B. admiration C. disbelief D. disgust
  • 21. Page 21 of 55 72. What is the sum of all the two digit numbers which are divisible by 5? A. 945 B. 950 C. 960 D. 1050 73. In an experiment, a vacuum is created when all air is removed from a tube. A coin and bits of confetti are released in the vacuum at the same time. They fall at the same rate and reach the bottom at the same time. The experiment proves that I. in a vacuum, the rate of acceleration is the same for all objects regardless of weight. II. outside a vacuum, air resistance is what makes different objects fall at different rates. III. gravity has no effect at all on objects that fall in a vacuum. A. I and II B. I and III C. I, II and III D. II and III 74. Mary Rose, an 18-year-old was sexually abused by 3 teenagers from well-to-do families from Makati. Despite pressures, she came out into the open to get justice. Which need did Mary Rose satisfy? A. Need for family unity B. Need for civic responsibility C. Need for universal solidarity D. Need for personal integrity 75. Below is the poem written by Edgar Lee Masters in 1915: What Happened to Tom Merritt? At first I suspected something She acted so calm and absent-minded. And one day I heard the back door shut, As I entered the front, and I saw him slink Back of the smoke house into the lot, And across the field. And I meant to kill him on sight.
  • 22. Page 22 of 55 But that day, walking near Fourth Bridge, Without a stick or a stone at hand, All of a sudden I saw him standing, Scared to death, holding his rabbits, And all l could say was,Don't, Don't, Don't, As he aimed and fired at my heart. Which of the following techniques is used in the poem? A. Verse B. Rhyme C. Free verse D. Personification 76. To pass her English Test, Lucille must get 75% of the items correct. Out of 80 questions, how many must she correctly answer? A. 55 B. 60 C. none of these D. 65 E. 70 77. 4 1/5 + 3 2/7 = __________ A. 7 3/12 B. 7 3/35 C. 7 17/35 D. 7 1/35 E. none of these 78. The principle under which a thermostat operates is the same when A. a gas expands to fill the container in which it is held. B. a pendulum swings when it is set into motion. C. a chemical reaction occurs when two substances combine. D. the level of mercury rises or falls in a glass tube.
  • 23. Page 23 of 55 79. Which is the BEST WAY to write the underlined portion of this sentence? A person should keep in mind some basic safety rules when you are deciding whether or not to use a fire extinguisher. A. rules you decide B. rules when you decided C. rules you are deciding D. rules when deciding 80. Which is the MOST important perceived need and problem of the Filipino family? A. Unemployment or financial problem B. Proneness to vices C. Over protectiveness of children D. Double standard on the roles of male and female 81. Which one BEST defines personal integrity? A. The unity between ignorance and reality B. The unity of man's deeds, words, thoughts and realities C. The unity of man's social, political and physical aspects D. The relationship between virtue and conduct 82. Which is the length of the hypotenuse of a right triangle with legs 5 inches and 12 inches? A. 17in. B. 13 in. C. 11in. D. 20 in. E. none of these 83. Ang sumusunod ay isang talumpati na may pamagat na SA KABATAAN na sinulat ni Onofre Pagsanghan. Isa sa mga salitang napag-aralan natin sa wikang Pilipino ay ang salitang NABANSOT. Kapang ang isang bagay daw ay dapat pang lumaki ngunit ito'y tumigil na sa paglaki, ang bagay na ito raw ay NABANSOT. Marami raw uri ng pagkabansot ngunit ay pinakamalungkot na uri raw ay ang pagkabansot ng isipan, ng puso at ng diwa.
  • 24. Page 24 of 55 Ang panahon ng kabataan ay panahon ng paglaki, ngunit ang ating paglaki ay kailangang paglaki, at pag-unlad ng ating buong katauhan, hindi lamang ng ating sukat at timbang. Kung ga-poste man ang ating taas at ga-pison man ang ating bigat ngunit kung ang pag-iisip naman nati'y ga-kulisap lamang kay pangit na kabansutan. Kung tumangkad man tayong tangkad-kawayan at bumilog man tayong bilog-tapayan, ngunit kung tayo nama'y tulad nibondyingay di mapagkatiwalaan-anong laking kakulangan. Kung magkakatawan tayong katawangTarzanat mapatalas ang ating isipang sintalas ng kay Rizal, ngunit kung ang ating kalooban nama'y itim na duwende ng kasamaan-anong kapinsalaan para sa kinabukasan. Kinabukasan, kabataan, tayo raw ang pag-asa ng Inang Bayan. Tayo raw ang maghahatid sa kanya sa langit ng kasaganaan at karangalan o hihils sa kanya sa putik ng kahirapan at kahihiyan. Ang panahon ng pagkilos ay ngayon, hindi bukas, hindi sa isang taon. Araw-araw ay tumutuwid tayong palangit or bumabaluktod tayong paputik. Tamang-tama ang sabi ng ating mga ninunong kung ano raw ang kinamihasnan ay siyang pagkakatandaan. Huwag nating akalaing makapagpapabaya tayo ng ating pag-aaral ngayon at sa araw ng bukas ay bigla tayong magiging mga dalubhasang magpapaunlad sa bayan. Huwag nating akalaing makapaglulublob tayo ngayon sa kalaswaaan at kahalayan at sa mahiwagang araw ng bukas sigla tayong magiging ulirang mga magulang. Kabataan, tunay na pag-ibig sa bayan, ang tunay na nasyonalismo, ay wala sa tamis ng pangarap wala rin sa pagpag ng dila. Ang tunay na pag-ibig ay nasa pawis ng gawa. Bakit di dapat tumangkad tulad nibondying? A. Di ito magpakakatiwalaan B. Di totoo ito C. May kakulangan ito D. Magulo kasi ito 84. Lucy's husband has been a drug dependent. She wanted him rehabilitated to be economically productive. Where will she commit her husband? A. DARN B. Bukang Liwayway Center C. DARE D. NFPI 85. Two days after Japan attacked Pearl Harbor, Roosevelt made the following statements: In the past few years and most violently in the past few days, we have learned a terrible lesson. We must begin the great task that is before us by abandoning once and for all the illusion that we can never again isolate ourselves from the rest of humanity. In the statement, Roosevelt is expressing the ideas of __________. A. an internationalist
  • 25. Page 25 of 55 B. an anti-imperialist C. an imperialist D. an isolationist 86. Which is the sum of the infinite progression 3/2, 1, 2/3, 4/9 . . . ? A. 6½ B. 5½ C. 4½ D. 7½ E. none of these 87. How much larger is the supplement of a 57 degree angle than the complementof a 75 qegree angle? A. 108 degrees B. 18 degrees C. 105 degrees D. none of these E. 123 degrees 88. Ang sumusunod ay isang talumpati na may pamagat na SA KABATAAN na sinulat ni Onofre Pagsanghan. Isa sa mga salitang napag-aralan natin sa wikang Pilipino ay ang salitang NABANSOT. Kapang ang isang bagay daw ay dapat pang lumaki ngunit ito'y tumigil na sa paglaki, ang bagay na ito raw ay NABANSOT. Marami raw uri ng pagkabansot ngunit ay pinakamalungkot na uri raw ay ang pagkabansot ng isipan, ng puso at ng diwa. Ang panahon ng kabataan ay panahon ng paglaki, ngunit ang ating paglaki ay kailangang paglaki, at pag-unlad ng ating buong katauhan, hindi lamang ng ating sukat at timbang. Kung ga-poste man ang ating taas at ga-pison man ang ating bigat ngunit kung ang pag-iisip naman nati'y ga-kulisap lamang kay pangit na kabansutan. Kung tumangkad man tayong tangkad-kawayan at bumilog man tayong bilog-tapayan, ngunit kung tayo nama'y tulad nibondyingay di mapagkatiwalaan-anong laking kakulangan. Kung magkakatawan tayong katawangTarzanat mapatalas ang ating isipang sintalas ng kay Rizal, ngunit kung ang ating kalooban nama'y itim na duwende ng kasamaan-anong kapinsalaan para sa kinabukasan. Kinabukasan, kabataan, tayo raw ang pag-asa ng Inang Bayan. Tayo raw ang maghahatid sa kanya sa langit ng kasaganaan at karangalan o hihils sa kanya sa putik ng kahirapan at kahihiyan. Ang panahon ng pagkilos ay ngayon, hindi bukas, hindi sa isang taon. Araw-araw ay tumutuwid tayong palangit or bumabaluktod tayong paputik. Tamang-tama ang sabi ng ating mga ninunong kung ano raw ang kinamihasnan ay siyang pagkakatandaan. Huwag nating akalaing makapagpapabaya tayo ng ating pag-aaral ngayon at sa araw ng bukas ay bigla tayong magiging mga dalubhasang magpapaunlad sa
  • 26. Page 26 of 55 bayan. Huwag nating akalaing makapaglulublob tayo ngayon sa kalaswaaan at kahalayan at sa mahiwagang araw ng bukas sigla tayong magiging ulirang mga magulang. Kabataan, tunay na pag-ibig sa bayan, ang tunay na nasyonalismo, ay wala sa tamis ng pangarap wala rin sa pagpag ng dila. Ang tunay na pag-ibig ay nasa pawis ng gawa. Sa alin makikita ang tunay na NASYONALlSMO? A. Diwa B. Gawa C. Sulat D. Salita 89. Alin sa mga salita ang kasingkahulugan ng salitang may salungguhit? Ang ama ni Anita ay kilalang bulanggugo sa kanilang lalawigan. A. Laging ibinubulong B. Laging handang gumasta C. Laging handang makipag-away D. Laging handang makipagtalo 90. What values are being given priority by juries in criminal cases? A. The rights of the criminal over the strict interpretation of the law B. The safety of the community over symphathy for the criminal C. The needs of the criminals over the advice of the judge D. The punishment of the criminal over the safety of the community 91. What correction should be made to this sentence? Recently, educators exammined the effectiveness of computer instruction in schools. A. Replace educators with educator's B. Change the spelling of exammined to examined C. Change schools to Schools D. Replace computer with computer's 92. Si Dr. Jose Rizal ay sumulat ng aklat ng itinampok sa iba't ibang bansa. Ang pangungusap ay nagagamit bilang __________?
  • 27. Page 27 of 55 A. panuring B. pamuno C. tuwirang layon D. paksa 93. A nation in which loess would likely be found today is A. Iceland B. United States C. Japan D. Ecuador 94. Which method of reproduction provides for the most variety of offspring? A. Cloning B. Sexual reproduction C. Asexual reproduction D. Cellular reproduction 95. Ano ang ipinahihiwatig ng salitang may salungguhit? Matayog ang lipad ni Pepe kaya't bata pa siya'y nagsisikap na siya. A. May kayabangan si Pepe. B. Mataas ang pangarap ni Pepe. C. Marunong si Pepe. D. Ibig ni Pepeng maabot ang langit. 96. Below is an excerpt from John F. Kennedy's "Inaugural Address." In your hands, my fellow citizens, more than mine, will rest the final success or failure of our course. Since this country as founded, each generation of Americans has been summoned to give testimony to its national loyalty. The graves of young Americans into answered the call to service surround the globe. Now the trumpet summons us again-not as a call to bear arms, though, arms we need, not as a call to battle, though embattled we are; but a call to bear the burden of a long twilight struggle, year in and year out, rejoicing in hope, patient in tribulation, a, struggle against the common enemies of man: tyranny, poverty, disease, and war itself. Can we forge against these enemies a grand and global alliance, North and South, East and West, that
  • 28. Page 28 of 55 can assure a more fruitful life for all mankind? Will you join me in this historic effort? In the long history of the world, only a few generations have been granted the role of defending freedom in its hour of maximum danger. I do not shrink from this responsibility; I welcome it I do not believe that any of us would exchange places with any other people or any other generation. The energy, the faith, the devotion which we bring to this endeavor will light our country and all who serve it, and the glow from that fire can truly light the world. And so, my fellow Americans, ask not what your country can do for you; ask what you can do for your country. My Fellow citizens of the world ask not what America will do for you, but what together we can do for the freedom of man. One of the purposes of the speech is to motivate listeners to A. serve their country B. enlist in the armed forces C. prepare themselves for battle D. preserve the right to bear arms 97. Which is the BEST evidence that helium gas is lighter than air? A. Helium has the lowest boiling point of all elements. B. Helium atoms do not combine with other air atoms. C. Helium-filled balloons rise in air. D. By volume, helium makes up only 0.0005% of air. 98. Spouses Jose S. Luz and Celerina Luz filed a petition to adopt Gregorio Luz Ona, their nephew. The spouses are childless and they reared Gregorio from his birth in 1971 until 1975 and they continued to support him. Gregorio had to be left in the Philippines when the spouses went to the United States where Jose is employed. The MSSD recommended the adoption to the court on the premise that petitioners are in a better position to provide for the minor child than the natural parents who are impoverished. What could be the possible decision of the court on the petition? A. The court denied the petition because the spouses are already aliens B. The petition for adoption was granted because the court finds that it is to the best interest of the child. C. The petition for adoption was denied because the spouses are non-residents of the Philippines. D. The court denied the petition on the premise that the trial custody requiredby PD 603 cannot be effected for spouses are non-residents 99. As a representative of the Urban Poor Commission of the Association of Religious Superiors (ARS), which action will you most likely take to resolve the long-term roots of structural inequalities- proliferation of child labor and child prostitution?
  • 29. Page 29 of 55 A. Raise views of human rights abuse. B. Organize regular programs for information and discussion of human rights. C. Conduct skills training. D. Raise questions over the government's commitment to rebuild human rights 100. The main message of the Moral Recovery Program launched by Leticia Ramos Shahani starts with the __________. A. world B. family C. self D. nation 101. The Miranda Family purchased a 250-pound side of beef and had it packaged. They paid Php365.00 for the side of beef. During the packaging, 75 pounds of beef were discarded as waste. What was the cost per pound for the packaged beef? A. Php2.08 per pound B. none of these C. Php2.50 per pound D. Php2.06 per pound E. Php2.30 per pound 102. Which has become a prototype of other schemes that defeated the real and true purpose of the CARL? A. The conversion of farmlands to industrial complexes B. The stock option scheme of Hacienda Luisita C. The conversion from agricultural to subdivision D. The voluntary offer to sell 103. A carpenter wanted three pieces of wood each 1 1/2 feet long. If he planned to cut them from a 6-foot piece of wood, how much of the piece would be left? A. 4 1/2 ft B. 1 1/2 ft C. 4 1/2 ft
  • 30. Page 30 of 55 D. 3 ft E. none of these 104. Which pitch is used for the word STRANGE? What a strange story! A. 3 B. 2 C. 4 D. 1 105. Which of the following methods can all diabetics control their condition and avoid heart disease and blindness? I. Regulating their intake of glucose II. Increasing the levels of insulin in the body by taking insulin injections III. Maintaining a reasonable exercise regimen to keep weight down. A. I B. II C. I and II D. I and III 106. How many gallons of water will fill a fish tank that is 18 inches by 12 inches by 48 inches? (There are 231 cubic inches per gallon). Round your answer to the nearest gallon. A. 45 gallons B. 40 gallons C. none ofthese D. 47 gallons E. 38 gallons 107. Which is the BEST WAY to write the underlined portion of this sentence? The hosts also benefit from running such a business because they can stay at home, make money, and meeting a variety of people. A. get to meet
  • 31. Page 31 of 55 B. to meet C. and meet D. and be meeting 108. New peonies, perennial plants that produce showy flowers can be propagated from the parent plant by dividing corns that grow underground. This reproductive form resembles a A. bulb B. runner C. seed D. bud 109. Alin sa mga sumusunod ang mensahe ng epiko ng IIokano na BIAG NI LAM-ANG? A. Pinatutunayan ng epiko ang yaman ng Ilokano sa lahat ng bagay. B. Kailangan paniniwalaan ang ukol sa bisa ng mga anting-anting dahil sa mga pangyayaring nagpapatunay dito. C. Dito nagpapatunay na walang kamatayan. D. Masasalat ang mga katutubong ugali at mga tradisyong dapat pagyamanin at panatilihin upang pakinabangan ng kabataan. 110. The Soviet Union's attempt to establish a missile base in Cuba is interpreted as a direct violation of A. the Truman DoCtrine B. the Monroe Doctrine C. the Strategic Arms Limitations Talks (SALT) D. the Declaration of Independence 111. What correction should be made to this sentence? Most fire-related death's result from household fires; yet many people do not have fire extinguishers in their homes. A. Remove the comma after fires B. Change result to results C. Change have to has D. Replace death's with deaths
  • 32. Page 32 of 55 112. Employees at Shaira's Musicmart get a 20% discount on all purchases. If Theresa buys three tapes at Php7.49 each, how much will she have to pay after her employee discount? A. Php16.98 B. Php19.98 C. Php18.98 D. Php17.98 E. none of these 113. Which family obligation is especially valued? A. Supplying groceries to relatives in remote barrios B. Providing health assistance to relatives living in the same locale C. Sending to college relatives in remote barrios D. Keeping immediate family members out of trouble 114. Which word in the passage does NOT require a change in pitch to show confidence? I am the master of my fate, I am the captain of my soul. A. am B. captain C. master D. fate 115. Mang Tacio has been unemployed for quite sometime due to his negative attitudes toward work. Which program of the Department of Social welfare and Services will help him? A. Income in Kind Program B. Anti-Medicancy program C. HRD Program D. Social Insurance Program 116. Scientist also find that other stalky vegetables such as carrots also help lower blood pressure. This statements is BEST classified as A. experiment B. nonessential fact
  • 33. Page 33 of 55 C. finding D. prediction 117. Alin uri ng parirala ang may salungguhit sa pangungusap? Utang sa kanyang sipag at sikap sa paggawa ang kanilang maalwang pamumuhay. A. Pangngalan B. Pangngalang-diwa C. Pawatas D. Pang-ukol 118. In which kingdom should MOLD be classified? A. Protista B. Plantae C. Fungi D. Animalia 119. What is meant by TWO PINS in this sentence? For two pins I could have hit him on the nose. A. A second course of action B. Without much persuading C. The second chance D. Have a second alternative choice 120. Which is the BEST WAY to write the underlined portion of this sentence? There is smoke detectors in many homes to warn residents of a fire, but fire extinguishers can actually help people fight fires. A. They're is B. Their is C. Their are D. There are
  • 34. Page 34 of 55 121. Alin sa mga sumusunod ang aral na ibinigay ng ANG ALAMAT NI MARIANG MAKILlNG na ikinuwento ni Jose Rizal? A. Pagyamanin at pangalagaan ang ating bayan at lahi pagka’t hiyas at yaman natin ito. B. Pag-ibig ang makapagbabago sa mundong ito. C. Kabanalan ang magpatawad at tulungan ang isang nagkasala. D. Dahil sa pagmamalabis at pagsasamantala, maraming biyaya ang sa kanya'y nawawala. 122. Which one should be TRUE if Earth's rotation axis did not tilt? I. Days and nights would be the same length everywhere on Earth. II. There would be no hours of darkness on points along the equator. III. Earth would have no seasons. IV. Each part of the Earth would have the same daily temperature pattern. A. II and IV B. III and IV C. IV and I D. I and III 123. The San Pascual Credit Cooperative of Quezon City wishes to appy for a loan of five million pesos from one of the financial institution, EXCEPT A. Development Bank of the Philippines B. Central Bank of the Philippines C. Philippine National Bank D. Land Bank of the Philippines 124. What is meant by SOFT OPTION in these sentences? Rebecca realized that if she stayed in her present job it would mean competing with an envious rival. Leaving the company would probably be a soft option. A. An action that is difficult to take B. An action that is not agreeable C. An action that is easier. D. An action that is weakly funded
  • 35. Page 35 of 55 125. Which of the following foreign policy actions today is a direct result of early American imperialist policy? A. Stationing of American troops in West Germany B. America's military support of Israel in the Middle East C. America's patrolling of waters off the Libyan coast D. America's establishment of military bases in the Philippines 126. Anong tayutay ang tinutukoy nito? Durog ang katawang bumagsak sa semento si Miguel. A. Pagtutulad B. Pagbibigay katauhan C. Pagmamalabis D. Pagwawangis 127. Mr. Garcia owns a 10½ hectare tract of land. He plans to subdivide this tract into ¼ hectare lots. He must first set aside 1/16 of the total land for roads. How many lots will this tract yield? A. 30 B. 35 C. 42 D. 45 E. none of these 128. The following are legitimate children, EXCEPT A. those born by artificial insemination. B. those legitimated. C. those born during a valid marriage of parents. D. those born out of a valid marriage of the parents. 129. The stanza below is taken from "Barter" by Sara Teasdale. Life has loneliness to sell, Music like a curve of gold, Scent of pine trees in the rain, Eyes that love you, arms that hold,
  • 36. Page 36 of 55 And for your spirit's still delight, Holy thoughts that star the night. To what does Teasdale compare music? A. The scent of pine trees B. A curve of gold C. Eyes that love D. The rain 130. Which is the BEST WAY to write the underlined portion of this sentence? The studies revealing that, for various reasons, girls spent less time working with computers' than boys. A. Revealing studies B. Studies revealed C. Studies' revelations D. Studies will reveal 131. Sabihin ang aspekto ng pandiwa sa pangungusap na ito. Mag-aral sa bahay ng mga araling ukol sa halaman. A. Pawatas B. Kontemplatibo C. Imperpektibo D. Perpektibo 132. 10 - 3 2/17 = __________ A. 7 2/1 7 B. 6 2/1 7 C. 6 1/17 D. 6 15/17 E. none of these 133. Below is a biographical sketch. Success often comes to those with humble beginnings, Elvis Aron Presley was born on January 8,
  • 37. Page 37 of 55 1935, in Tupelo, Mississippi. He first sang in a church choir and taught himself to play the guitar, but he never learned to read music. By 1953, he had moved to Memphis, Tennessee, graduated from high school, and enrolled in night school to become an electrician. That year, at Sun Records, Presley recorded a personal record for his mother, a song that was heard by the company's president. As a result of the president's recognition, Presley's first record,That's all Right, Mama,was out in 1954. He toured the South, and in 1955 five of his records were released simultaneously. His first national television appearance was that year on Jackie Gleason'sThe Stage Show,but Presley became known for his appearance onThe Ed Sullivan Show,where the young singer gyrated as he sangrock n' rollmusic. During the live television performance, Presley was photographed only from the waist up because his motions were considered obscene. Elvis the Pelvisbegan his film career in 1956 with LOVE ME TENDER and signed a long term film contract. The movie critics were not always kind, but teenagers flocked to Presley's films. Within a few short years, Presley had established a career that would span twenty-five years of ups and downs and make him one of the most popular entertainers in history. Long after his untimely death at age 42, Presley would be remembered asThe King of Rock n' Roll The statement thatsuccess often comes to those with humble beginningswould apply best to which of the following figures? A. Ramon Magsaysay B. Gloria M. Arroyo C. Corazon C. Aquino D. Joseph E. Estrada 134. At which time during the year does the ozone level present a particular health threat in urban areas for people with respiratory problems? A. Spring B. Fall C. Summer D. Winter 135. Why was San Andres Cooperative Association of Paco, Manila not exempted from taxation? A. It accumulated reserves and undivided net savings of Php8,000,000.00 B. It accumulated reserves and undivided net savings of Php10,000.00 C. It accumulated reserves and undivided net savings Php9,000,000.00 D. It accumulated reserves and undivided net savings of Php11,000.00 136. If a farmer would want assistance like pricing, guarantee for all agricultural produce or cooperative management training, where would he go?
  • 38. Page 38 of 55 A. Support Services of the Department of Agrarian Reform B. Special Agrarian Court under the Regional Trial court C. DAR Adjudication Board D. Land Bank 137. Which of the following represent, ethnocentric behavior? A. A tourist who lectures his foreign hosts on the "uncivilized" nature of their marriage customs B. A student who tutors an immigrant in English C. A Hispanic community group demands that public aid forms be published in English and Spanish D. A peace Corps volunteer who helps dig wells in Central Africa 138. Which word is pronounced with a /z/ at the end? A. Maps B. Jakes C. Laughs D. Buys 139. Below is the poem written by Edgar Lee Masters in 1915: What Happened to Tom Merritt? At first I suspected something She acted so calm and absent-minded. And one day I heard the back door shut, As I entered the front, and I saw him slink Back of the smoke house into the lot, And across the field. And I meant to kill him on sight. But that day, walking near Fourth Bridge, Without a stick or a stone at hand, All of a sudden I saw him standing, Scared to death, holding his rabbits, And all l could say was,Don't, Don't, Don't, As he aimed and fired at my heart. The poet introduces the poem with lines 1-3 to show us that Tom Merritt A. had suspected that his wife was seeing another man
  • 39. Page 39 of 55 B. was sure that his wife was ill C. was a very suspicious person D. was sure that his wife still loved him 140. Which of the following procedures used by a farmer is NOT related directly to preventing erosion? A. Contour plowing around a hill B. Planting more seeds than are necessary to yield a bountiful crop. C. Planting grass in gullies to act as a filter D. Planting crops in alternate rows (strip farming) 141. Which is the BEST WAY to write the underlined portion of this sentence? However, their VCR kept them from missing their favorite primetime shows. A. keepes B. keeps C. had kept D. keeped 142. Below is a biographical sketch. Success often comes to those with humble beginnings, Elvis Aron Presley was born on January 8, 1935, in Tupelo, Mississippi. He first sang in a church choir and taught himself to play the guitar, but he never learned to read music. By 1953, he had moved to Memphis, Tennessee, graduated from high school, and enrolled in night school to become an electrician. That year, at Sun Records, Presley recorded a personal record for his mother, a song that was heard by the company's president. As a result of the president's recognition, Presley's first record,That's all Right, Mama,was out in 1954. He toured the South, and in 1955 five of his records were released simultaneously. His first national television appearance was that year on Jackie Gleason'sThe Stage Show,but Presley became known for his appearance onThe Ed Sullivan Show,where the young singer gyrated as he sangrock n' rollmusic. During the live television performance, Presley was photographed only from the waist up because his motions were considered obscene. Elvis the Pelvisbegan his film career in 1956 with LOVE ME TENDER and signed a long term film contract. The movie critics were not always kind, but teenagers flocked to Presley's films. Within a few short years, Presley had established a career that would span twenty-five years of ups and downs and make him one of the most popular entertainers in history. Long after his untimely death at age 42, Presley would be remembered asThe King of Rock n' Roll The author uses the phrase ups and downs to refer to Presley's
  • 40. Page 40 of 55 A. gyrations as he performed B. increasing and decreasing finances C. successes and disappointments in his career D. use of drugs,uppers and downers 143. In an experiment, a drop of blue ink is placed on the surface of a glass of water. In a few minutes, the drop of ink is dispersed throughout the water, turning it light blue. The result of the experiment proves that A. molecules of ink and molecules of water are in constant motion. B. heat causes the ink to disperse. C. a new compound is formed by the combination of ink and water. D. ink molecules have less density than water molecule. 144. An elderly woman suffered a stroke, a restriction of blood flow to the brain. If the stroke caused to the right side of her body to become temporarily paralyzed, she most likely experienced a decreased blood flow to A. the left side of her body. B. the front of her brain. C. the left side of her brain. D. the right side of her brain. 145. What is the function of DIFFUSION in the human body? A. Regulates blood flow B. Plays an insignificant role in the body's functioning C. Allows an even distribution of substances throughout all cells of the body D. Comes into play in times of extreme illness 146. Each of the following objects is designed to employ the buoyancy principle EXCEPT a A. life preserver B. kite C. submarine D. canoe
  • 41. Page 41 of 55 147. It was also discovered that the chemical 3m butylphthalide can lower the blood pressure of rats. This statement is classified as A. prediction B. experiment C. finding D. nonessential fact 148. Lines 11 and 12 are taken from the poem "maggie and milly and molly and may." For whatever we lose (like a you or a me) it's always ourselves we find in the sea Which of the following ideas is the author expressing? A. The sea is a source of life and death. B. The sea represents all of our moods. C. The sea is the best place for a person to reflect about life. D. The sea and its surroundings can give people a fresh view on life. 149. Which refers to FUNNEL EFFECT? A. The belief that every criminal gets caught and is punished. B. The belief that crime is under control in the United States. C. The idea that only a very few suspects arrested for committing a crime are actually punished. D. The idea that all crimes are put into the same criminal justice system. 150. Alin ang naayong pamagat sa tanagang sinulat ni Jose Villa Panganiban? Ano man sa daigdig, Maaring magamit, Ano mang masaisip; Di sukat maiipit. A. Pagkainip B. Paraya C. Pag-asa
  • 42. Page 42 of 55 D. Pagbibigay 151. The undeclared war in Korea most closely resembled the situation of A. The Spanish-American War B. The Vietnam War C. World War I D. World War II 152. Alin ang di-karaniwang anyo ng pandiwang HINTAY KA? A. Tay B. Tayka C. Intay D. Teka 153. Below is the poem written by Edgar Lee Masters in 1915: What Happened to Tom Merritt? At first I suspected something She acted so calm and absent-minded. And one day I heard the back door shut, As I entered the front, and I saw him slink Back of the smoke house into the lot, And across the field. And I meant to kill him on sight. But that day, walking near Fourth Bridge, Without a stick or a stone at hand, All of a sudden I saw him standing, Scared to death, holding his rabbits, And all l could say was,Don't, Don't, Don't, As he aimed and fired at my heart. Who is the speaker of this poem? A. Tom Merritt B. Merritt's wife
  • 43. Page 43 of 55 C. God D. The sheriff 154. A car dealer is offering a rebate of Php7,500.00 on any new-car purchase. If the purchase price of a car is Php200,000.00 more than it was last year, what is the rate of discount offered by the rebate? A. 10% B. not enough information is given C. 7.5% D. 13.3% E. 14.2% 155. Which of the following internal forces interrupt the external forces of erosion? I. Forces that cause volcanoes II. Forces that cause ocean trenchers III. Forces that create mountains A. I, II and III B. I and III C. II and III D. I and II 156. Of the following, which is imposed a final tax of ten percent (10%)? A. PCSO and lotto winnings B. Books, literary works and musical compositions C. Currency bank deposit D. Royalties 157. What is meant by the expression TO GET BLOOD OUT OF A STONE in these sentences? Geraldo has owed me fifty thousand pesos for over a year now. I’ve asked him for it on several occasions, but it’s like trying to get blood out of a stone. A. Something is impossible. B. Someone refuses to cooperate.
  • 44. Page 44 of 55 C. Someone is willing to give what is asked. D. Someone wants revenge. 158. Four mangoes cost P29.00. At that price what will 2½ dozen mangoes cost? A. Php217.50 B. Php188.50 C. Php348.50 D. Php870.00 E. none of these 159. Which kingdom should STREPTOCOCCUS be classified? A. Protista B. Plantae C. Fungi D. Monera 160. Which of the following should you expect to be true about the rate of cellular respiration for a group of students who are the same age, height, and weight? A. Athletes would tend to have higher rates of cellular respiration than non-athletes. B. Africans would have a higher rate of cellular respiration than Asians. C. Boys would have a higher rate of cellular respiration than girls. D. Non-athletes would have higher rates of cellular respiration than athletes. 161. Without the process of meiosis, we can infer that offspring from sexual reproduction would A. have a high degree of genetic variety. B. have twice the assigned number of chromosomes. C. be identical. D. have a number of mutations. 162. You buy a new refrigerator for Php12,800.00 and make a down payment of Php2,500.00. If you finance the remainder at 8% annually for three years, how much will you actually pay for the refrigerator?
  • 45. Page 45 of 55 A. Php12,190.00 B. Php10,300.00 C. none of these D. Php12,772.00 E. Php15,272.00 163. Alin sa mga sumusunod ang aral na ibinibigay ng epikong Muslim na INDARAPATRA AT SULAYMAN? A. Pagmamahal B. Katapangan C. Katapatan, D. Pagtanaw ng utang-na-loob 164. Which of the following facts support the Big Bang theory's explanation of the creation of the universe? A. The universe does not expand nor contract. B. The universe seldoms expand. C. The universe will have background radiation. D. The universe has no beginning nor end. 165. Which are the next three terms in the progression 1/125, -1/25, 1/5 . . . 7 terms? A. -2, 6, -26 B. -3, 7, -27 C. -4, 8, -28 D. -1, 5, -25 E. none of these 166. What correction should be made to this sentence? First-borns often pattern their behavior after they're parents and other adults. A. Replace their with they're B. Change pattern to patterned
  • 46. Page 46 of 55 C. Replace they're to their D. No correction is necessary 167. The poem below is entitledSuburban Prophecywhich is written by Howard Nemerov. On Saturday, the power-mowers' whine Begins the morning. Over this neighbourhood Rises the keening, petulant voice, begin Green oily teeth to chatter and munch the cud. Monsters, crawling the carpets of the world, Still send from underground against your blades The roots of things battalions green and curled And tender, that will match your blades with blades Till the revolted throats shall strangle on. The tickle of their dead, till straws shall break Crankshafts like camels, and the sun go down On dinosaurs in swamps. A night attack Follows and by the time the Sabbath dawns All armored beasts are eaten by their lawns. To what does the phraseyour bladesin line 8 refer? A. Lawnmowers B. Roots C. Carpets D. Monsters 168. How much topsoil is needed to cover a garden 25 feet by 40 feet to a depth of 6 inches? A. 480 ft3 B. 440 ft3 C. 500 ft3 D. 460 ft3 E. none of these 169. Which word contains the /ae/ sound? A. Carriage B. Cabin
  • 47. Page 47 of 55 C. Caste D. Can 170. Which will solve poverty caused by capitalism? A. Fascism B. Communism C. Empirism D. Socialism 171. Which one is the human right to dignity? A. Political independence B. Honor and reputation C. Form associations D. Social and economic reforms 172. What correction should be made to this sentence? Buying a fire extinguisher knowing how to use it, and placing it in a location familiar to all family members can help protect families against fire. A. Insert a comma after extinguisher B. Change the spelling of families to familys C. Change placing to place D. Insert a comma after help 173. Which verb in the sentence is pronounced with the ending as /d/? They laughed and joked as they walked and played. A. Joked B. Laughed C. Walked D. Played 174. What correction should be made to this sentence?
  • 48. Page 48 of 55 Most State tourism departments and some travel agencies have bed and breakfast listings. A. Insert a comma after agencies B. Change have to has C. Change tourism to Tourism D. Change State to state 175. Which theory was asserted by the Pan-German belief in the superiority of the Aryan race and that the strength of the German culture came from a strong, healthy and rustic lineage? A. DependencyTheory B. Culture of Poverty Theory C. Social Darwinist Theory D. Theory of Capitalism 176. The hypotenuse of a right triangle is 25 feet. If one leg is 24 feet, what is the length of the other leg? A. 5 ft B. 5 ft C. 20 ft D. 7 ft E. none of these 177. Where would you commit a drug dependent for him to achieve a natural, tensionless, and anxiety-free state? A. NFPI B. DARN C. DARE D. Bukang Liwayway Center 178. Which is the equivalent common fraction of the repeating decimal 3.242424...? A. 107/33 B. 110/33 C. 109/33
  • 49. Page 49 of 55 D. 108/33 E. none of these 179. What is meant by AT SIXES AND SEVENS in this sentence? We moved into the house last week, but I'm afraid everything, is still at sixes and sevens. A. The things have not been shipped. B. In a state of confusion C. In an orderly manner D. The boxes are still intact 180. What is suggested in the opening line? June 13, 1986 - they came from all over America - 200,000 heroes strong, with their families. A. The writer holds great admiration for the veterans B. The writer was a veteran of the war C. The writer is opposed to the Vietnam War D. The writer is a flag-waving patriot 181. Which law of force and motion explains this occurrence, when a rocket is propelled upward by the powerful downward discharge of exhaust gases? A. Universal Gravitation B. Action and Reaction C. Applied Force D. Inertia 182. Tides, caused by the moon's gravity, create a frictional force that is gradually slowing down Earth's rotation speed. One million years from now, scientist may discover that compared to today, Earth's A. day is shorter B. year is longer C. day is longer D. year is shorter
  • 50. Page 50 of 55 183. Which area of the brain controls feeling on the left side of a person's face? A. The left occipital lobe B. The left parietal lobe C. The right parietal lobe D. The right occipital lobe 184. Which location should have most nearly twelve hours of daylight and twelve hours of darkness during December? A town that is located A. halfway between the Equator and the South Pole. B. close to the Equator. C. close to the North Pole. D. close to the South Pole. 185. During a recent shopping spree, Tomas and Nena bought some new accessories for their apartment. Nena chose a crocheted throw pillow at Php24.95, and Tomas purchased a rural landscape painting for Php135.00. How much did they actually spend if they paid 7% sales tax on their purchases? A. Php171.15 B. Php139.25 C. none of these D. Php159.95 E. Php148.75 186. Dadalaw sa mga paaralan si Dr. Filemon S. Salas, ang tagapamanihala ng mga paaralang lungsod, sa lungsod ng Pasay. Ang pangungusap ay nagagamit bilang A. panuring B. paksa
  • 51. Page 51 of 55 C. tuwirang layon D. pamuno 187. Ang sumusunod ay isang talumpati na may pamagat na SA KABATAAN na sinulat ni Onofre Pagsanghan. Isa sa mga salitang napag-aralan natin sa wikang Pilipino ay ang salitang NABANSOT. Kapang ang isang bagay daw ay dapat pang lumaki ngunit ito'y tumigil na sa paglaki, ang bagay na ito raw ay NABANSOT. Marami raw uri ng pagkabansot ngunit ay pinakamalungkot na uri raw ay ang pagkabansot ng isipan, ng puso at ng diwa. Ang panahon ng kabataan ay panahon ng paglaki, ngunit ang ating paglaki ay kailangang paglaki, at pag-unlad ng ating buong katauhan, hindi lamang ng ating sukat at timbang. Kung ga-poste man ang ating taas at ga-pison man ang ating bigat ngunit kung ang pag-iisip naman nati'y ga-kulisap lamang kay pangit na kabansutan. Kung tumangkad man tayong tangkad-kawayan at bumilog man tayong bilog-tapayan, ngunit kung tayo nama'y tulad nibondyingay di mapagkatiwalaan-anong laking kakulangan. Kung magkakatawan tayong katawangTarzanat mapatalas ang ating isipang sintalas ng kay Rizal, ngunit kung ang ating kalooban nama'y itim na duwende ng kasamaan-anong kapinsalaan para sa kinabukasan. Kinabukasan, kabataan, tayo raw ang pag-asa ng Inang Bayan. Tayo raw ang maghahatid sa kanya sa langit ng kasaganaan at karangalan o hihils sa kanya sa putik ng kahirapan at kahihiyan. Ang panahon ng pagkilos ay ngayon, hindi bukas, hindi sa isang taon. Araw-araw ay tumutuwid tayong palangit or bumabaluktod tayong paputik. Tamang-tama ang sabi ng ating mga ninunong kung ano raw ang kinamihasnan ay siyang pagkakatandaan. Huwag nating akalaing makapagpapabaya tayo ng ating pag-aaral ngayon at sa araw ng bukas ay bigla tayong magiging mga dalubhasang magpapaunlad sa bayan. Huwag nating akalaing makapaglulublob tayo ngayon sa kalaswaaan at kahalayan at sa mahiwagang araw ng bukas sigla tayong magiging ulirang mga magulang. Kabataan, tunay na pag-ibig sa bayan, ang tunay na nasyonalismo, ay wala sa tamis ng pangarap wala rin sa pagpag ng dila. Ang tunay na pag-ibig ay nasa pawis ng gawa. Alin salita ang paulit-ulit na binabanggit ni Onofre Pagsanghan? A. Nabansot B. Bayan C. Kabataan D. Kung 188. In the progression 18, -12, 8 . . . which term is 512/729? A. the 8th B. the 6th C. the 9th D. the 7th E. none of these
  • 52. Page 52 of 55 189. Why does a bullet when discharged into the air eventually fall to the ground? This is due to the Law of A. Universal Gravitation B. Applied Force C. Inertia D. Action and Reaction 190. Kilalanin ang uri ng pariralang may salungguhit. Ang pangangalaga sa mga likas na yaman ay tungkulin nating lahat. A. Pangngalan B. Pangngalang-diwa C. Pang-ukol D. Pawatas 191. If P75.00 is shared among three children in the ratio of 3:7:15. The size of the smaller share is A. Php9.00 B. Php15.00 C. Php25.00 D. Php35.00 E. none of these 192. To gain the attention of the audience, the trick is __________? A. start low, speak hurriedly B. start high, speak loudly C. start high, speak rapidly D. start low, speak slowly 193. Which is the BEST WAY to write the underlined portion of this sentence? Recently psychologists have been researching birth order, their research suggests that personality and intelligence are based partly on where a child ranks in the family. A. order. Their
  • 53. Page 53 of 55 B. order and their C. order, or their D. order their 194. Anong aral ang ibinibigay ng sumusunod na salawikain? Ang taong napapailalim, ay naipapaibabaw rin. A. Maaring ngayon ay hirap pagdating ng bukas ay may ginhawa rin B. Tiyak ang pag-unlad kapag nauna ang hirap C. Kung ano ang ibig natin ay mangyayari D. Magtiis kung dumarating ang hirap 195. Carolina Diaz filed a petition for habeas corpus against Mr. and Mrs. Ramon Alde to recover custody of Lina Diaz Tan aliasGracia Alde,the natural daughter of Carolina Diaz, who was a hostess. What could be the possible action of the court on the petition filed by Carolina Diaz? A. Her petition would be granted because she now works as a clerk in a prestigious office. B. Her petition would be denied because she was a former hostess. C. Her petition would be denied because when Gracia was given to the Aldes it was tantamount to abandonment of the child, resulting to termination of parental authority. D. Her petition would be granted because she is the natural mother. 196. Which is the main goal of drug abuse education? A. Arrest B. Prevention C. Control D. Rehabilitation 197. Why did the register of deeds charge Lucio Cruz registration fee the instrument relative to his loan? A. His loan was Php30,000.00 B. His loan was Php60,000.00
  • 54. Page 54 of 55 C. His loan was Php50,000.00 D. His loan was Php40,000.00 198. The poem below is entitledSuburban Prophecywhich is written by Howard Nemerov. On Saturday, the power-mowers' whine Begins the morning. Over this neighbourhood Rises the keening, petulant voice, begin Green oily teeth to chatter and munch the cud. Monsters, crawling the carpets of the world, Still send from underground against your blades The roots of things battalions green and curled And tender, that will match your blades with blades Till the revolted throats shall strangle on. The tickle of their dead, till straws shall break Crankshafts like camels, and the sun go down On dinosaurs in swamps. A night attack Follows and by the time the Sabbath dawns All armored beasts are eaten by their lawns. The poet's use of words such as whine, voice, teeth, chatter and munch is to suggest that the power- mowers are A. very powerful B. alive C. like cows D. green 199. Which explains the reason why there are continuous and increasing human rights violations? A. The United Nation's General Assembly approved only resolutions on human rights and the basic freedoms which are not binding. B. The solutions used are ineffective. C. The United Nations as an international body is rather slow in the exercise of its powers. D. The United Nations uses a single solution on all forms of human rights violations. 200. What is meant by LAST DITCH in this sentence? The aged bishop was prepared to fight to the last ditch to defend his good name. A. One's last courage
  • 55. Page 55 of 55 B. One's last strategy C. One's last hope D. One's last defense